Вы находитесь на странице: 1из 72

APARATUL RESPIRATOR

1. Definiți termenii de lobul pulmonar primar, acin pulmonar și lobul pulmonar secundar
:
Lobulul pulmonar primar
• Deservit de ultima bronhiolă respiratorie
• Format din ducte alveolare, saci alveolari și alveole

Lobulul pulmonar secundar este unitatea funcţională a plămânului


• Deservit de bronhiola terminală
• Măsoară 1-2.5 cm diametru
• Sunt delimitate de septurile interlobulare, la randul lor continue cu interstiţiul peribronhovascular
(ţesutul conjunctiv axial) și pleura (ţesutul conjunctiv periferic)
• Conţine 10-12 acini pulmonari și 30-50 lobuli pulmonari

Acinul pulmonar (unitatea respiratorie terminala)


• Deservit de prima bronşiola respiratorie
• unitatea pulmonara situata distal de bronsiola terminală
• Contine 100 ducte alveolare și 2000 alveole

2. Definiți cele 5 procese respiratorii :

a. Ventilația pulmonară– pătrunderea aerului prin căile respiratorii până la alveolele


pulmonare
b. Respirația externă – schimburile gazoase (O2 și CO2) între alveolele pulmonare și sângele din
capilarele pulmonare prin intermediul membranei respiratorii
c. Transportul gazelor – transportulO2 și CO2între plămâni și celulele organismului prin
intermediul sistemului cardio-vascular și al sângelui
d. Respirația internă – schimbul gazos dintre sângele capilarelor sistemice și celulele
organismului
e. Respirația celulara – procesul care folosește O2 și reactiile lanțului respirator mitocondrial al
celulei

3. Definiți noțiunile de semn, simptom, sindrom, cu exemple :


• Simptom
• Element subiectiv, observat de pacient si care nu poate fi observat/masurat direct
– De exemplu, durerea este un simptom (doar persoana care o experimenteaza o poate
resimti)
• Un semneste observabil/evaluabil de ceilalti
Tusea este un simptom, dar si semn (oricine o poate confirma)
• Simptomele si semnele sunt in general non-specifice, dar adeseori combinatia lor sunt puternic
sugestive pentru un anumit diagnostic
• In alte cazuri, ele sunt atat de specifice incat pot fi patognomonice.
*Un Sindrom :
• Ansamblu se simptome si semne clinice si paraclinice corelate intre ele care apar in cursul mai
multor boli
• In unele cazuri, un sindrom este atat de strans corelat cu o patogenie sau o etiologie, incat
cuvintele sindrom / boala au sfarsit prin a fi utilizate intersanjabil
– Acest lucru este valabil in special in sindroamele genetice.
– De exemplu, sindromul Down are o patogenie cunoscuta, astfel incat reprezinta un unic set
de simptome si semne , in ciuda nomenclaturii de “sindrom”.

4. Definiți următoarele: tuse, strănut, sputa:


Tusea : Reflex brusc și adesea repetitiv care permite îndepărtarea de la nivelul căilor respiratorii
inferioare a mucusului care contine particule straine/iritante și a virusurilor/microbilor.

Sputa: Secreții anormale (prin volum sau compoziţie) care provin din aparatul respirator bolnav.
*Aceste secreții au origine sub-glotică, fiind exteriorizate prin efortul de tuse.
*Nu trebuie confundate cu saliva sau cu secrețiile nazo-faringiene

5. Definiți următoarele: dispnee, durere:


Dispneea : este o respiratie dificila
Durera :osenzatie neplacuta localizataintr-o zona acorpului care reprezinta perceptiaunei leziuni existente

6.Ce urmărim la inspecția generală a pacientului?


• Vizualizarea exhaustiva si negrabita a pacientului
• Nu presupune doar examenul vizual
– Ascultati orice sunet care provine de la pacient
– Observati orice miros care vine de la pacient

7.Semnele detresei respiratorii :


Respiratie suieratoare(wheezing) sau respiratie muncita
Repspiratie anormala caracterizata prin demonstrarea obiectiva a eortului respirator
Hiperpnee , tahipnee , tiraj , utilizarea muschii inspiratori accesori

8.Ortopneea: definiție, semnificație clinică:

Definiție:
• O dispnee care apare în clinostatism.
• Invers, aceasta dispnee dispare atunci când pacientul se ridică.
• Provine din grec. orthos (pozitie ridicata) și pneo (respitatie) (adica respirație în pozitie verticală)

Semnificatie clinice :
• insuficienței ventriculare stângi și/sau al edemului pulmonar (95% din cazuri).

DAR SI IN :
bolile pulmonare : pneumonia, paralizia diafragmatică bilaterală, epanșamente pleurale astm bronsic si
poate fi intalnita la pacienti fara afectare cardiac sau pulmonara (ascita severa , obezitatea , paralizie
nerv frenic)
IN BPCO : RESPIRATIA SE AMELIOREAZA IN ORTOSTATISM.

9.Dispneea paroxistică nocturnă: definiție, semnificație clinică:


Definitia:
Dupa 1-2 ore de somn, pacientul se trezește brusc, se ridică în șezut, își lasă picioarele la marginea patului,
se ridică în picioare și deschide fereastra pentru a prinde ceva aer proaspăt, iar după câteva minute se
simte mai bine, suficient încât sa poată merge înapoi în pat.Bineînțeles, poziția verticală și nu aerul
proaspăt este responsabilă de dispariția DPN
Semnificatia clinica :
DPN este un semn frecvent al insuficienței ventriculare stângi.
DPN poate apare și la pacientii pulmonari

10. Platipneea: definiție, semnificație clinică:


Definitie:pacienții cu platipnee respiră mai bine când sunt întinși în pat(e opusul ortopneei)
Semnificatia clinica :Platipneea se asociază frecvent cu ortodeoxia, care este o desaturare a oxigen-
hemoglobinei în momentul ortostatismului.

11.Trepopneea: definiție, semnificație clinică:


Def:Este o respirație caracterizată prin incapacitatea pacientului de a sta culcat (sau ridicat) și preferința
pentru poziția de decubit lateral.motiv (crestere perfuziei )
Semif clin :atelectazia pulmonară unilaterală , insuficiență cardiacă congestivă , cu tumori endobronșice.

12. Tahipneea: definiție, semnificație clinică:


Def = accelerarea ritmului respirator (>25/min). De obicei, indică o boală cardio-respiratorie severă, care
impune o creștere compensatorie a efortului respirator. La pacientii spitalizați, prognosticul este rău
(prezice stopul cardio-respirator).
– Non spitalizati : pneumonie
– Embolia pulmonară
13. Bradipneea: definiție, semnificație clinică:
Def: = încetinirea ritmului respirator (<8/min). Oboseala din obstrucția severa a căilor aeriene, sedarea,
hipertensiune intracraniană, supradozare de opiacee, insuficiența tiroidiană.
Semn clin: Hipotiroidie(dar poate sugera și o afectare a sistemului nervos central)

14. Apneea: definiție, semnificație clinică:


Def : Absența respirației cel putin 20 de secunde la pacientul în stare de veghe și cel putin 30 de secunde
la pacientul în stare de somn
Semn clin :Se întâlnește adesea la pacienții cu disfuncție neuromusculară (apnee centrala) sau obstrucție a căilor
aeriene indusă de perioadele de somn Rapid Eye Movement (REM) (sleep apneea de tip obstructiv).

De observat totuși că apneea este și evenimentul final al tuturor insuficiențelor respiratorii, de cauză pulmonară sau
neuromusculară.

15. Hiperpneea: definiție, semnificație clinică:


Def : hiperpneea este o respiratie rapidă și adâncă unde creste volumul current.
Hiperpneea este un mechanism compensator in acidoze(hiperventilatie)
Exista o diferenta intre hiperpnee si hiperventilatie in boli cardio vasculare. In bolile cadio vasc respiratia e
superficiala.

16. Hipopneea: definiție, semnificație clinică:


Def:respirația superficială
Semn clin :De obicei, indică sindromul obezitate-hipoventilație (sindrom Pickwick), dar și iminența
insuficienței respiratorii. Hipopneea se asociază adesea cu perioade de apnee.

17. Respirația Cheyene-Stokes: definiție, semnificație clinică,


consecințe:
Def:Este o respiratie periodica, un Model anormal de respirație caracterizat de un ciclu crescendo-
descrescendo (hipopnee-hiperpnee-hipopnee) al volumului curent, urmat de o oprire temporară a
respirației (apnee).
Ciclul are durata de 30 sec → până la 2 minute.
Perioada apneică este de obicei mai scurtă.
Semn clin : insuficiența cardiacă congestivă
afecțiuni neurologice (meningite, infarcte/hemoragii bilaterale/unilaterale sau leziuni traumatice ale
trunchiului cerebral/supratentoriale
la oamenii sănătoși
la vârstnici
în somn (central sleep-apnea)
consecinte :Consecințele respirației Cheyne-Stokes
Modificări severe ale fluxului sanguin cerebral, determinate de alternanța de flux cerebral mare
(hiperpnee) și flux cerebral scăzut (hipopnee).
Aceste modificări sunt probabil responsabile de unele modificări de status cerebral descrise la acesti
pacienți: alertă, agitație, creșterea tonusului muscular în timpul hiperpneei, urmate de somnolență,
imobilitate și scăderea tonusului în timpul apneei.

18. Respirația Biot: definiție, semnificație clinică


Respirația Biot este o variantă a respirației Cheyne-Stokes
Caracterizată printr-o succesiune de hiperventilatie și apnee, dar fără pattern-ul tipic crescendo-
descrescendo; de asemenea, nu este caracterizată prin regularitate.
De asemenea, este mai rară.
Apare de obicei la pacienții cu meningită sau compresie cerebrală.
În consecință, are un prognostic (efect) mai rău, ducând de obicei la apnee completă și stop cardiac.

19. Respirația apneustică: definiție, semnificație clinică:


Def :Este un pattern particular al respirației, caracterizată prin o fază inspiratorie adâncă (“gasp”) urmată
de o perioadă de apnee și un expir rapid.
Semn clin: Este tipică pentru leziunile pontine sau bulbare

20. Hipervențilația centrală: definiție, semnificație clinică:


Def :Este un continuum de hiperpnee + tahipnee (adica respirații adânci și rapide).
Nu are niciun fel de legatură cu alte hiperventilații (cum ar fi respirația Küssmaul, care, oricum este mai
puțin rapidă, dar mai adâncă)
Semn clin : Se întâlnește la pacientii cu leziuni ale mezencefalului sau pontin superioare.

21. Ventilația ataxică: definiție, caracteristici, semnificație clinică:


Def: Ventilația ataxică: grec. a- (lipsa a) și taxis (ordine) sau agonică (gasping respiration, agonal breathing)
Este un ritm respirator total anarhic (neregulat) – un fel de fibrilație a centrilor respiratori, cu deplasări
continue du-te-vino de la hiper- la hipo-ventilație și de la hiper- la hipo-pnee intercalate de perioade de
apnee.
Respirație anormală caracterizată prin : Respirație muncită , “gasping” , Vocalizări stranii , Mioclonii
Este o situație medicală extrem de serioasă, impunând atenție medicală imediată, întrucât în general
semnalează decesul iminent, evoluând către apnee completă.
Durata – de la doua respirații la cateva ore
Apare la pacienții cu leziuni ale bulbului.De aici, termenul de respirație agonică (agonală).

22. Respirația stertoroasă: definiție, semnificație clinică


Def: Un sunet „gâlgâitor-grohăitor” produs de pacienții prea bolnavi pentru a scăpa de secrețiile
respiratorii.
Semn clin:Pe vremea lui Laënnec, respirația stertoroasă era un semn de pneumonie severă în faza de
iminență a parezei mușchilor respiratori și a morții.

23. Respirația cu ,,buzele strânse”: definiție, semnificație clinică:


Def: Data fiind hiperinflația alveolară (și scăderea elasticității pulmonare) caracteristică BPOC, pacienții
sunt expuși riscului de închidere a cailor respiratorii și captare de aer.
În acest moment, organismul recurge la expirul „cu buzele strânse”, ca și cum ar umfla un balon.
Acest tip de expir crește presiunea în interiorul căilor aeriene, astfel inducând un auto-PEEP (presiune
pozitiva tele-inspiratorie). Se asocieaza cu wheezing.
semn clin: bpco , emfizem

24. Paradoxul abdominal: definiție, semnificație clinică, metodă de


detectare:
Def: În timpul respirației normale, peretele toracic și cel abdominal sunt sincronizate atât în inspir cat si in expir. In
paradoxul abdominal toracele și abdomnul devin asincrone. Torace spre sus si abdomen spre jos.

Semn clin: oboselii diafragmatice

Metoda de detectare a paradoxului abdominal:


*Palparea bimanuală* :
Puneți o mână pe toracele pacientului și cealaltă pe abdomen, apoi urmăriți deplasările acestora.
Bineînțeles, paradoxul poate fi detectat și prin inspecție.
Acesti pacienti sut ortopneici (in ortostatism continutul abdomenului aplica o presiunea mai mare asupra
diafragmului).

25. Respirația asincronă: definiție, semnificație clinică:


Def:Este o forma specială de paradox abdominal, observat la pacientii cu BPCO .
În acest caz, mișcarea în interior a peretelui abdominal observată în expirul precoce este imediat urmată
de o mișcare către exterior(ba3d ma ynzal l abdomen bl expir bysla5 haydik l t3leye)
semn clin: un prognostic rău, fiind un predictor al insuficienței respiratorii, al necesității de ventilație
mecanică și al decesului

26. Respirația alternantă: definiție, semnificație clinică:


Def:Pacienții cu alternanță respiratorie prezintă utilizarea alternă a diafragmului și a mușchilor intercostali,
cu toracele și abdomenul deplasându-se mai întâi într-un fel, apoi invers.
Semn clin: Este un alt semn al slăbiciunii mușchilor respiratori
pacienții pot trece de la respirație alternantă la paradox abdominal și invers

27. Respirația în peritonită: caracteristici generale:


În timpul respiratiei, peretele abdominal ramane imobil, deci respirația este limitata.
Aceasta limitare este:
difuză în peritonita generalizată și
localizată în afectarea focală.
De exemplu, în diverticulită, aria imobilă se află în cardanul inferior stâng al abdomenului.
În apendicită, aria imobilă se afla în cadranul inferior drept.

28. Asterixis: definiție, semnificație clinică, metodă de detectare:


Def: Tremurul bilateral , Pacientul cu asterixis are o incapacitate de a menține o poziție prin tonusul
muscular. Pierderea bruscă a tonusului la încheietura mâinii determină o flexie ritmică a mâinii
Manevra :
1. Cereți pacienților să ridice brațele, să dorsiflexeze încheieturile mâinilor, și sa răsfire degetele - ca în
cazul în care încearcă să opreasca traficul. Aceasta provoacă, de obicei, "fluturări" la încheietura mâinii.
sau
2. hiperextindeți ușor încheieturile mâinilor.

semn clin: encefalopatia hepatică ("tremor hepatic") și encefalopatia renală


Retenția de CO2 – insuficiența respiratorie tip 2 (de ex. BPOC)
Dezechilibrul electrolitic (hipokaliemie, hipomagneziemie, hipoglicemie).

29. Hipocratismul digital: definiție, semne și simptome, metodă de


detectare:
Def:O deformare a degetelor și unghiilor asociată cu un număr de boli, mai ales ale inimii și plămânilor.
Semne si simptome : 5 etape :
1. Fără hipocratism vizibil. Fluctuația și înmuierea patului unghial (creșterea “flotabilității”).
2. Hipocratism ușor. Pierderea unghiului normal <165 ° (unghiul Lovibond) dintre patul unghial
și restul falangei.
3. Hipocratism moderat. Creșterea pulpei degetelor și pozitivarea testului Schamroth
4. Hipocratism sever. Îngroșarea întregii falange distale (care seamănă cu un băț de tobă)
5. Aspect lucios și striatii ale unghiilor și pielii
Metoda de detectare:
1.Mediusul fixează falanga a 2-a, iar policele fixează articulația interfalangiană
2.Degetele arătătoare balotează baza unghiei dintr-o parte în alta.
30. Hipocratismul digital: cauze principale, generale, cu câte 2
exemple:
Cauze:
1.cardio-vascualre: mixomul atrial , cardiopatiile congenitale cianogene
2.respiratorii:cancerul bronhopulmonar , fibroza pulmonara (nu bpco)
3.gastro-intestinale: malabsorbtia , ciroza hepatica
4.altele : boala gaves , hipocratism digital familial (africa)

Un stidiu belgian in 2008 a indicat : 1% din cazuri

31. Osteoartropatia hipertrofică parapneumică: semnificație clinică:


Este O formă specială de hipocratism digital
Se asociază de obicei asociat cu cancer pulmonar (HPOA secundar)
HPOA primar: - HPOA fără semne de boală pulmonară
-anomalii cardiace
-scăderea de prostaglandină E2 .

32. Cianoza: diferențiere între cianoza centrală și cea periferică:


Def:Colorație albăstruie a tegumentelor și mucoaselor determinată de o concentrație crescută a
hemoglobinei reduse (dezoxihemoglobinei) (>5g/dL)
Clasificare:
Cianoza centrală:
Sângele pleacă cu saturația scăzută de la nivelul complexului cardio-pulmonar.
Periferia este bine irigată – cianoză caldă.
Cianoza periferică:
Sângele se desaturează în periferie din cauza irigației insuficiente – cianoză rece.

33. Cianoza centrală: cauze, semne:


Cauze:
Altitudine înaltă
Orice afecțiune respiratorie – obstructive, restrictive sau vasculare
Afecțiuni cardiace congenitale cu șunt dreapta stânga
Semne:
Localizare:limba si mucoasa orala
Extremitati: calde
Volumul pulsului: crescut
Dispnee:prezenta

34. Cianoza periferică: cauze, semne:


Cauze:
Expunere la frig
Obstrucție arterială/venoasă
Reducerea debitului cardiac – insuficiență cardiacă, șoc + Toate cauzele de cianoză centrală
Semne:
Distal: varful nasului , varful degetelor , extremitati
Extremitati:reci
Volumul pulsului: scazut
Dispnee: absenta

35. Sindromul venei cave superioare: cauze, tablou clinic:


Cauza:
90% este neoplazică
Carcinomul bronhogen
Limfomul malign
Tabloul clinic :
Dispnee , Cefalee , Edem al feței, membrelor superioare, gâtului (gulerul lui Stokes)
Distensie venoasă la nivelul gâtului, toracelui superior, membrelor superioare
Semnul Pemberton (congestie facială la ridicarea brațelor)

36. Inspecția gâtului – tehnica de examinare a presiunii venoase


centrale, valori normale:
Lumina trebuie să fie tangențială pentru a face să apară lumini și umbre.
Pacient cu trunchiul ridicat la 45°.
Gâtul nu trebuie să fie flexat.
Folosind o riglă centimetrică, măsurați distanța verticală dintre unghiul Louis (articulația manubrio-
sternală) și cel mai înalt nivel al pulsațiilor venei jugulare. O a doua riglă care o intersectează pe cea dintâi
poate fi de ajutor.
Măsurați înălțimea venelor gâtului de la un punct 0 situat la nivelul unghiului sternal (metoda Lewis).
Adăugați 5 cm la măsurare, deoarece atriul drept este la 5 cm sub unghiul sternal.
PVC normala ≤ 8 cm H2O
Notă: Măsurarea presiunii venoase jugulare va fi dificilă dacă pulsul este> 100 pe minut.

37. Inspecția gâtului - semnul Kussmaul, semnificație clinică:


Creșterea paradoxală a PVC în inspir
indică imposibilitatea de umplere a ventriculului drept.
Semn clin:
pericardita constrictivă
cardiomiopatia restrictivă
epanșamentul pericardic
insuficiența cardiacă dreaptă severă.

38. Inspecția gâtului – mușchii accesori : enumerare, semnificație


clinică:
M.accesori :Sternocleidomastoidienii , cei trei m.scaleni , m.pectorali mic
Muschi care intervin in inspir : diafragm , intercostali externi , sternoceidomastodian , scaleni , pectorali ,
trapez , dintat anterior si marele dintat.(in mare parte diafragm).
Muschi care intervin in expir: m.abdominali , intercostali interni , m.dintat postero-inferior.(in mare parte
reculul pasiv).
Semn clin:Mișcarea ascendentă a claviculei cu 5 mm suplimentari este un semn valoros al unei boli
obstructive severe, corelându-se cu o FEV1 de 0.6 L
Bpco: retractiile inspiratori ale foselor suprasternale si supraclaviculare.

39. Anomaliile formei toracelui – coloana vertebrală:


Anomalii în planul sagital:
Creșterea concavității – lordoza
Creșterea convexității – cifoza
Anomalii în planul frontal:
Sunt curburi laterale ale coloanei – scolioze
Consecințele sunt în primul rând respiratorii – și adesea sunt dramatice:
În cazul în care sunt suficient de severe, anomaliile de coloană pot schimba mecanica ventilației,
determinând hipoventilație localizată. Ca urmare, dezechilibrul V/Q rezultat determină hipoxemie locală →
vasoconstricție pulmonară → hipertensiune pulmonara si, in cele din urma → insuficientă cardiacă
dreaptă.

40. Anomaliile formei toracelui – stern – pectux excavatus:


Este o adâncitură sternală profundă, caracterizată prin deplasarea înapoi a 1/2 (2/3) inferioare a sternului,
adeseori mai vizibilă dupa inspir profund.
Frecv: 1 la 300-400 de nașteri.
Semn clin : tahiaritmii , Suflu la orificiul pulmonar , crește frecvența infecțiilor pulmonare și chiar astm
bronsic

41. Anomaliile formei toracelui – stern – pectus carinatum:


Toracele în carenă (toracele de porumbel, pectus carinatum) , Este opusul toracelui excavat , cu
proiectarea înainte a sternului.(apare la 0.06% nașteri)
Semn clin: durere ,fatigabilitatea precoce , crește frecvența infecțiilor pulmonare și chiar astm bronsic
(datorita hipoventilatie) nu există nici sufluri, nici aritmii.

42. Anomaliile formei toracelui – coastele – modificări de pantă și de


formă:
Normal , Raportul dintre diametrul anterp-posterior si transversal este 0.70-0.75
La pacienții cu emfizem, bronșită cronică sau status astm , tracțiunea excesivă exercitată de muschii
respiratori accesori asupra coastelor duce la egalizarea diametrelor sau cel putin 0.90.
aceasta determină aparitia toracelui în butoi. Deci toracele in butoi este aproape rotund cu coastele
aproape orizontalizate si raportul intre cele 2 diametre aproape unitar.
Semnificația clinică: emfizem , bpoc , batrani.

43. Șanțul lui Harrison: aspect, cauze:


Aspect: Pieptul semana cu corpul unei viori
Cauza: tractionarea de catre diafragma a coastelor slabite de rahitism sau o alta boala osoasa .

44. Semnul lui Hoover: aspect, cauze:


Este o mișcare inspiratorie paradoxală a cutiei toracice laterale la pacientii cu BPOC.
Normal – rebordurile costale se mișcă foarte puțin în timpul inspirației liniștite, dar, dacă o fac, ele tind să
se miste spre exterior și în sus.
Anormal (BPCO) – în timpul inspirului, rebordurile costale se mișcă înspre interior.
Explicația – diafragmul este aplatizat, dar funcțional, iar contracția lui trage înspre interior coastele
inferioare.

45. Palparea toracelui – trahee: metodă ( explicați o singură metodă),


semnificație clinică:
1.Cereți pacientului să se ridice la marginea patului, să se aplece puțin în față și să păstreze capul drept.
2.Cu două degete de o parte și alta a traheei
Indexul și mediusul
sau
cu cele doua police
3.Comparați adâncimile celor doua fose. Acestea ar trebui să fie simetrice. Dacă nu, pacientul are o
deplasare traheală – de obicei spre partea cu fosa mai mică.

46. Palparea toracelui – trahee: cauzele deplasării traheale, diagnostic


diferențial între cauze:
Deplasare inspre partea afectata(ipsilaterala) :atelectazie : - pneumoectomie / fibroza pulmonara
Deplasare impotriva partii afectate(controlaterala): epansament oleural voluminos: pneumotorax sub
tensiunea
Alte deplasari: mase madiastinale
Diagnostic diferential:
Atelectazia – celelalte semne ale examenului fizic (matitate, scăderea vibrațiilor vocale și silențiul
auscultator) sunt ipsilaterale.
Epanșamentul pleural masiv – celelalte semne ale examenului fizic (matitate, scăderea vibrațiilor vocale și
silențiul auscultator) sunt controlaterale.

47.Semnul Oliver:
O deplasare în jos a cricoidului care coincide cu fiecare bătaie a inimii.
Pentru a detecta mai bine aceasta deplasare, cereți pacientului sa stea cu cu capul și bărbia în extensie,
apoi prindeți osul cricoid. Aplicați o presiune ascendentă blândă cu degetul mare și arătătorul.
O deplasare în jos a traheei, sincronă cu fiecare sistolă, sugerează prezența unui anevrism al arcului aortic,
care trece pe deasupra bronhiei principale stângi și o apasa în jos la fiecare contractie cardiaca.
Cu toate acestea semnul poate apare la pacientii cu tumori mediastinale sau chiar BPOC simplu

48. Semnul Cardarelli:


Un alt semn de anevrism aortic, un fel de semn al lui Oliver provocat.
Pentru a obține semnul Cardarelli, apăsați pe cartilajului tiroidian, deplasandu-l usor spre stânga
pacientului.
Această deplasare a traheei crește contactul dintre bronhia principala stânga și aortă, făcând astfel posibil
ca examinatorul să se simtă o pulsație transversală a traheei, în cazurile de anevrism al arcului aortic.

49. semnul campbell:


O deplasare în jos a cartilajului tiroidian în timpul inspirației.
Deși, de asemenea, o formă de deplasare a traheei, semnul Campbell nu se datorează anevrismului aortic,
ci obstructiei cronice.
Pentru a obține semnul Campbell, puneți vârful degetului arătător pe cartilajul tiroidian, și căutați o
miscare de coborâre a traheei în timpul inspirației.
Acest semn este un predictor precis al severității obstructiei fluxului de aer, corelandu-se bine cu durata
simptomelor și reducerea volumul expirator fortat în 1 secunda (FEV1).Totusi, semnul Campbell nu este
specific pentru BPOC, deoarece este pozitiv și în alte situații de detresă respiratorie acută. De fapt, acesta
se datorează unei tracțiuni inspiratorii excesive asupra traheei în timpul contracției diafragmatice.

50. Vibrațiile vocale: semnificația clinică a creșterii și a scăderii intensității


lor cu exemple:
Creșterea VV = consolidare
Scăderea VV = epanșament pleural, pneumotorax, empiem.
Valoarea VV constă în faptul ca pot aduce dovezi suplimentare dacă alte elemente ale examenului fizic
pulmonar sugerează prezența unui proces patologic.
De exemplu, în pneumonie, VV variază în funcție de tipul acesteia: alveolară/bronhoalveolară.
•Dacă pneumonia este limitată la alveole (adica bronhii patente înconjurate de alveole pline cu puroi), VV
vor fi mai intense, întrucăt lichidele (dar și solidele de la acest nivel) transmit sunetul mai bine decât aerul.
•În schimb, dacă alveolita se extinde la bronhii (bronhopneumonie, tipică pentru Haemophilus influenzae),
transmiterea VV va fi temperată de secrețiile intrabronșică, și astfel, VV vor fi mai slabe.
•Prin urmare, pneumonia crește VV numai dacă este de tip alveolar; ea scade VV dacă este
bronhoalveolară

51. Percutia indirecta, principii generale si unde se percuta


Metodă introdusă de Pierre Adolphe Piorry (1826):
Un plexor lovește un pleximetru aezat pe corp.Mediusul mâinii dominante (plexor) lovește mediusul mâinii
non-dominante (pleximetru).
Se foloseste in practica în scop comparativ și topografic.
Percutați întotdeauna dinspre rezonanță înspre matitate.În general, percuția poate fi efectuată doar pe
toracele posterior. Cu toate acestea, în cazul in care auscultația revelează o anomalie pe pereții laterali sau
anteriori, percuția pe aceste arii poate ajuta la identificarea cauzei anomaliei.

Varianta maximalistă – 26 de puncte: torace A, L, P;


Varianta intermediară (clasică) – 24 de puncte: torace A si P;
Varianta simplificată – 14 puncte: torace A si P;
Varianta minimă – 8 puncte: torace P
52. Sunetele percuționale, caracteristici, exemple patologice
Timpanism: normal in spatiul Traube, patologic in pneumotorax;
Hiperrezonantă: patologic: plamanul emfizematos;
Submatitate : normal:ficat; patologic: pneumonie lobara, revarsat pleural, pahipleurita;
Matitate: normal: coapsa; patologic: epansament pleural;

53. Linia Damoiseau – Ellis, triunghiul Grocco-Rauchfuss și triunghiul Garland


1. Linia Damoiseau-Ellis
•Un parametru semiologic percuțional care desemnează pe spatele toracelui un epanșament pleural.
•Linie curbă cu convexitate superioară, care pornește de la coloana toracală , se înalță către linia axilară
posterioară sau medie, de unde începe să coboare înspre anterior.
2. Triunghiul paravertebral Grocco-Rauchfuss
•O arie de submatitate paravertebrală triunghiulară determinată de deplasarea mediastinală.
3. Triunghiul Garlad
•Zona hipersonoră situată între coloana vertebrală și curba Damoiseau-Ellis.

54. Percuția auscultatorie: metodă, semnificație clinică


Medicul loveste ușor cu vârful degetului manubriul sternal, în același timp ascultând cu stetoscopul peste
locații simetrice ale peretelui toracic posterior.
•Teoria este că, la pacientul sănătos, sunetul astfel generat călătorește nestingherit prin plămâni și ajunge
la peretele toracic opus într-o manieră simetrică.
•Astfel, orice asimetrie în intensitatea sunetului este considerat un semn de afecțiune pulmonară.
•Prin utilizarea acestei tehnici, Guarino a fost capabil sa detecteze leziunile pulmonare <2 cm în diametru
(care sunt aproape imposibil să fie identifice prin percuția convențională).
•Cu toate acestea, studii efectuate pentru a confirma aceasta tehnica au avut rezultate contradictorii.

55. Definiți caracteristicile sunetelor: frecvență, amplitudine, timbru


Frecvența undei = caracterizeaza inaltimea sunetului. Sunetele cu frecventa inalta sunt ascutite, iar cele cu
frecventa joasa sunt sunete grave.
Frecvența fundamentală (primară) – cea mai joasă frecvență a undei sunetului și determină înălțimea
finală a acestuia.

Amplitudinea undei = caracterizeaza taria sunetului. Sunetele cu amplitudine mare sunt sunete puternice,
iar cele cu amplitudine mica sunt sunete slabe.
Timbrul = Diferențiază între două sunete cu aceeași tărie și înălțime.

56. Mecanismele de producere ale sunetelor pulmonare


Căile aeriene mari centrale (trahee, bronhii principale) =>Flux de aer turbulent
•Moleculele de aer se ciocnesc întâmplător una de cealaltă și de pereții căilor aeriene.
•Această mișcare a aerului determină sunetele respiratorii.
Căile aeriene medii =>Flux de aer vorticular
•Ramificările dihotomice succesive separă fluxul aerian în pături cu viteze diferite , a căror interacțiune
genereaza vârtejuri (vortexuri).
•Această mișcare a aerului este adesea denumită mixtă (sau tranzițională), întrucât seamană și cu fluxul
laminar și cu cel turbulent.

57. Sunetele de bază: enumerare, origine


Sunet normal al căilor aeriene mari (tubar):
– Origine : faringe, laringe, trahee, bronhii mari;
– Mecanism de producere: Flux turbulent;
– Caracterizeaza configuratia cailor aeriene superioare;

Sunet pulmonar normal (murmur vezicular):


– Origine: inspir -> bronhii lobare-segmentare; expir -> bronhii principale-lobare
– Mecanism: flux turbulent si vorticular, mecanisme necunoscute;
– Relevant pentru ventilatia regionala;
58. Zgomotele traheal, bronșic, bronho-vezicular: localizare normală și
raport inspiro-expirator:
Sunetul bronsic (suflu tubar):
– Localizare: pe trahee si de o parte si alta a acesteia;
– Inaltime: inalta (100-1500 Hz);
– Taria: Puternic; (I > E)
– Timbrul: rezonant, aspru, gaunos;
– Raport I-E: pauza intre ele
59. Stridorul
Sunet supradăugat muzical, ascuțit, inspirator, se aude la nivelul căilor aeriene superioare sau la distanță,
chiar fără stetoscop. Indică întotdeauna obstrucția căilor aeriene superioare.Este mai puternic la nivelul
gâtului, este întotdeauna inspirator.
–Stridorul supraglotic – sucționarea pliurilor ari-epiglotice în interiorul căilor aeriene în timpul inspirului
–Stridorul glotic – determinat în special de paralizia corzilor vocale
–Stridorul subglotic – apare la copiii cu inflamație virală a laringelui (laringita)

60. Sunetul pulmonar normal (murmurul vezicular ): localizare, raport


inspiro-expirator.
În momentul în care traheea pătrunde în torace (și începe să se ramifice în bronhii și apoi bronhiole), căile
respiratorii sunt înconjurate de o mantie de alveole, care acționează ca un amortizor, scăzând intensitatea
și frecvența sunetelor tubulare inițiale.
–Aerul alveolar acționează ca equalizer-ul unui echipament stereo
•Elimină undele cu frecvență înaltă (>250Hz) și intensitate mare.
•Prezervă undele cu frecvență joasă (200-250Hz) și intensitate slabă.
–Sunetele care se aud în periferie sunt generate în bronhii
•Inspir: bronhii lobare-segmentare
•Expir: bronhii pincipale-lobare
Pe măsură ce sunetul CRS se apropie de suprafața toracelui, își schimbă caracteristicile și devine sunetul
respirației pulmonare (sunetul pulmonar normal, murmurul vezicular), al oamenilor sănătoși.

61. Sunet pulmonar anormat ( crescut în intensitate – ex. )


62. Sunet pulmonar anormal ( scăzut în intensitate – ex. )

63. Suflul tubar: cauzele apariției, caracteristici ( înălțime, tărie, timbru,


raport inspiro-expirator ):

Mai are o caracteristică – se asociază întotdeauna cu pectorilocvia afonă;


Semnifică pierderea capacității plămânilor de a acționa ca filtru pentru frecvențele înalte fie prin inlocuirea
aerului alveolar (ex lichid) sau prin colapsul pulmonar(epansament pleural masiv)

64. Suflu cavernos: semnificație clinică


–Sunet bronșic cu frecvență joasă.
–Apare în prezența unei cavități pulmonare (pe vremea lui Laënnec, aproape întotdeauna cavernă Tbc, dar
și în abcese golite de conținut sau cavități bronșiectatice).
65. Suflu amforic: semnificație clinică
–Sunet bronșic cu tonuri cu frecvență de bază joasă, dar cu supratonuri înalte și cu timbru metalic.
–Apare în prezența unei cavități pulmonare
•Superficiale
•De cel puțin 5cm
•Cu bronhie patentă
•Cu pereți subțiri

66. Crepitantele: definiție, mecanism de producere, momentul apariției în


ciclul respirator
Sunt cele mai utile sunete supraadăugate din cauza asocierii puternice dintre momentul în ciclul
respirator și sediul producerii sunetului.
În funcție de momentul de apariție în cursul ciclului respirator: Proto-
inspiratorii, Mezo-inspiratorii, Tele-inspiratorii, Inspiro-expiratorii.
Mecanismul de aparitie al crepitantelor:
–Afectări ale interstițiului pulmonar → presiune interstițială mare
 Infiltrarea lichidiană interstițială (puroi, sânge, transudat).
 Fibroze interstițiale → creșterea reculului elastic
–Presiunea interstițială mare → colabarea expiratorie a căilor aeriene.
–Redeschiderea lor bruscă în inspir determină un crepitant tele-
inspirator.
–Întrucât presupun infiltrarea interstițială, apar preponderent la bazele
pulmonare
–Nu se curăță cu tusea (întrucât nu se datorează secrețiilor).

67. Crepitantele expiratorii – semnificație clinică


90% dintre crepitante apar în inspir, dar 10% apar mezo- și tele-expirator.

 La pacienții cu patologie obstructivă (bronșită, bronșiectazii)


– groase, proto-expiratorii, frecvente. Scad în număr dupa tuse.
 La pacienții cu patologie restrictiva (pneumopatii interstitiale fibrozante)
– fine, mezo- sau tele-expiratorii, nu dispar dupa tuse.
– numărul crepitantelor se coreleaza cu reducerea capacității de difuziune, deci cu severitatea bolii.

68. Sibilanțele: definiție, mecanism de producere, momentul apariției în


ciclul respirator
Sunete supraadăugate continue (durată lungă ≥50msec), muzicale, preponderent expiratorii (dar pot
apare și în inspir). Uneori pe toata durata ciclului respirator, dar niciodată nu sunt exclusiv inspiratorii.
Este comparat cu suieratul vantului pe sub usa.
Mecanismul de aparitie:

 Vibrațiile apar atunci când fluxul de aer trece rapid prin căile aeriene cu diametrul interior îngustat.
– Bronhospam
– Secreții intrabronșice
– Edem peribronșic (insuficiența cardiacă stângă)
– Tumora bronșică parțial obstructivă
 Mecanismul are la bază principiul Bernoulli
– Fluxul de aer care trece rapid prin o bronhie îngustată va “sucționa” în interior peretele aerian,
creând în aces

În funcție de frecvența fundamentală (primară) – 4 tipuri


Sibilante – frecvența fundamentală >400Hz.

– Sibilante polifonice – conțin mai multe note, toate începând și sfârșind în același moment (Astmul
bronșic)
– Sibilante monofonice multiple (Astmul bronșic sever)
– Sibilante monofonice – notă unică (Tumora bronșică care obliterează aproape complet o bronhie)

Ronflante – frecvența fundamentală <200Hz. t fel o vibrație și, astfel, un sibilant.

69. Sibilantele propriu-zise: timp de apariție, semnificație clinică


În clinică

– Preponderent expiratorii (pot apare și în inspir, dar niciodată doar în inspir).


– Tipice pentru astmul bronsic, dar pot fi auzite și la persoanele sănătoase la capătul unui expir
prelungit.
– Reflectă alterări extinse ale căilor respiratorii; în consecință, sunt prezente pe arii pulmonare
extinse.
70. Sibilante monofonice multiple: semnificație clinică
În clinică
– Caracteristice pentru astmul bronșic sever (status asthmaticus), unde apar pe întregul torace.
– Pot să apară în expir, deși tind să acopere (mai tipic) intregul ciclu respirator.
71. Ronflantele: timp de apariție, semnificație clinică
Comparat cu Sforăitul.Apar in inspir si in expir.

În clinică
– Bronșită, bronșiectazii
– Modificările de postură pot să le atenueze sau să le elimine.
– Reflectă procese difuze, deci se auscultă pe arii extinse.
72. Țipătul de pescăruș tele-inspirator(Squawk)
Sunet supradăugat scurt, muzical, inspirator.
Mecanism
– Este produs de redeschiderea cailor aeriene parțial colabate si, de aceea, sunt specifice pneumopatiilor
interstitiale (mai ales la nivelul bazelor), unde pot coexista cu crepitantele tele-inspiratorii.
– În acest caz, căile aeriene au un lumen foarte neregulat, din cauza regenerării mucoasei (ca în
bronsiolitis obliterans). Aceasta neregularitate (și îngustare consecutivă) este responsabilă de frecvența
înaltă și de timbrul specific.
– Astfel, aerul care trece prin calea aeriana recent redeschisă, dar încă parțial îngustată (din cauza
lumenului neregulat) va determina atât un crepitant (întrucât calea aeriana se deschide abrupt) cât și
un sibilant (cand aerul navalește prin strâmtoare).

Caracteristici
– Cel mai adesea sunt inspiratorii, unice (monofonice), scurte, muzicale, cu frecvență inalta și
determina un timbru foarte caracteristic, care seamănă cu un unic sibilant tele-inspirator
(Laënnec”: le cri d’un petit oiseau).

În clinică

– Cele mai frecvente cauze: fibroza pulmonară, alveolita alergică și bronșiolita obliterantă.

73. Frecătura pleurală: definiție, mecanism de producere, momentul


apariției
Sunet supraadăugat non-muzical, scurt, exploziv, inspiro-expirator, mai frecvent la baze.
Un tip special de sunete pulmonare supradaugate:
– Puternic
– Inspiro-expirator – segmentul inspirator este mai puternic, segmentul expirator este mai lung

Mecanism
– Frecarea care apare între cele două foițe pleurale inflamate
– Cele doua foițe devin aspre prin inflamație și prn faptul că sunt acoperite de fibrină.
– Aceasta determină un sunet ca de piele uscată, în mod caracteristic inspiro-expirator.
74. Diferența între frecătura pleurală și crepitante

75. Bronhofonia
Manevra
–Cereți pacientului să rostească tare “33” sau “99” în timp ce auscultați plămânii (în maniera sistematică
decrisă anterior).
Rezultat, interpretare
Normal: Vocea normală transmisă este moale, asurzită și indistinctă. O puteți auzi cu stetoscopul, dar nu
puteti ințelege exact ce spune pacientul.

– Plămânul normal filtrează frecvențele înalte (ceea ce face ca vocalele sa nu se mai auda).

Patologic: Bronhofonie = auziti un “33” (sau “99”) clar.

– Plămânul lipsit de aer transmite frecvențele înalte ale vorbirii


– Înseamnă că densitatea pulmonară este anormal crescut (consolidare).

76. Pectorilocvia afonă


Manevra
–Cereți pacientului să șoptească “33” sau “99” în timp ce auscultați plămânii (în maniera sistematică
decrisă anterior).
Rezultat, interpretare
Normal: Vocea șoptită nu prezintă frecvențele înalte, care sunt filtrate selectiv prin plămân; în consecință,
vocea șoptită este asurzită și aproape inaudibilă.
Patologic: În caz de consolidare, vocea șoptită este transmisa clar

77. Egofonia
Manevra
– Cereti pacientului sa rostească “eeeeeeee” în timp ce auscultați plămânii (în maniera sistematică
decrisă anterior).

Rezultat, interpretare
– Normal: In mod normal, sunetul “eeeeeeee” se aude pe intreaga arie pulmonara.
– Patologic: La nivelul ariilor de condensare, sunetul “ee-ee-ee-ee” se transforma în “aa-aa-aa-aa”.
78. Sucusiunea hipocratică
Manevra

– Prin metoda percuției, alegeți partea care contine aer.


– Plasați stetoscopul la acest nivel.
– Medicul clatină pacientul de o parte și alta.

Rezultat
– În caz de hidro- sau pio-pneumotorax, se aude un sunet hidro-aeric.

79. Semnul Hamman


–Sunet cu caracter de cracment auscultabil la nivelul precordiului, între spațiile intercostale 3-5, sincron cu
bătăile inimii.
–Extrem de sensibil în caz de pneumomediastin
Mai sensibil decât radiografia pulmonară.

80. Semnul d’Espine


Manevra
– Auscultați de o parte și de alta a vertebrelor toracice superioare și comparați intensitatea și calitatea
sunetelor la acelasi nivel.

Rezultat

– În mod normal, sunetele veziculare de o parte și de alta a coloanei sunt mai puternice decât cele
auscultate pe coloana la același nivel.
– Semnul d’Espine este pozitiv atunci când sunetul auscultat pe coloană este mai puternic decât cele
auzite de o parte și de alta a coloanei.

Interpretare
– Semnifică prezența unei mase la nivelul mediastinului posterior (cel mai adesea limfadenopatie).

81. Compartimentare mediastin anatomic și radiologic


Are 4 compartimente anatomice:
Împărțit în compartimentele superior și inferior printr-o linie orizontala imaginară care unește
articulația manubrio-sternală și marginea inferioară a T4.
Compartimentul inferior este subdivizat în
– Mediastinul mediu – conține pericardul și conținutul său, ca și vasele mari și căile aeriene mari;
– Mediastinul anterior – între mediastinul mediu și stern;
– Mediastinul posterior – între mediastinul mediu și coloana vertebrală.
Compartimentarea radiologica:

1. Metoda Felson se bazează pe RgCP laterală


– Linie care merge de la diafragm către apertura toracică superioară prin
spatele inimii și anterior de trahee, separând mediastinul anterior de cel
mediu;
– Linie care unește punctele aflate la 1.5cm în spatele marginilor anterioare
ale corpilor vertebrali, separănd mediastinul mediu de mediastinul
posterior.

2. Compartimentarea anatomo-radiologica:
A.Compartimentarea anatomică: superior, anterior, mijlociu
și posterior.
B.Schema alternativă, preferată de radiologi, omite
compartimentul superior.
– Aria situată posterior de stern și anterior de cord și vasele
mari (săgeți albastre) definește mediastinul anterior.
– De asemenea, o linie care urmează marginea posterioară a
traheei și a inimii (săgeți galbene) separă mediastinul
mediu de cel posterior.
– Mediastinul posterior este delimitat posterior de corpurile vertebrale (steluțe roșii).
82. Conturul cardio-mediastinal drept
Conturul cardio-mediastinal drept – de sus în jos:
–Marginea paratraheală dreaptă (vasele mari)
•Observabilă în 2/3 din filmele normale
•Conține vena brahio-cefalică dreaptă și vena cavă superioară
–Arcul venei azygos
•La pacienții vârstnici, aorta ascendentă se proiectează adesea la dreapta venei cave superioare.
–Vena cavă superioară (VCS)
–Atriul drept
–Vena cavă inferioară (VCI)

83. Conturul cardio-mediastinal stâng


Conturul cardio-mediastinal stâng – de sus în jos:
–Marginea paratraheală stângă: Conține artera carotidă comuna
stânga, artera subclavie stângă și vena jugulară stângă
–“Butonul aortic”- reprezintă marginea laterală stângă a aortei în
timp ce se arcuieste deasupra bronhiei principale stângi și a vaselor
pulmonare.
–Artera pulmonară
–Auriculul atriului stâng
–Ventriculul stâng
*Deplasarea sau pierderea definiției acestor linii poate indica o patologie (cum ar fi un anevrism sau o
consolidare pulmonară subiacentă).

84.

Conturul cardio-mediastinal anterior-de sus în jos:


–Mediastinul superior
•Vasele mari
•Timusul
–Aortaascendentă
–Conul de ejecție a ventriculului drept
–Ventricululdrept

85. Conturul cardio-mediastinal posterior - de sus în jos:


–Atriul stâng și venele pulmonare
–Atriul drept
–Vena cavă inferioară.
86.

Analiza imaginii radiologice pentru: proiecție, includere, artefacte – criterii


de pozitivitate
Principalele proiectii: Rg PA si Rg Laterala.
Principii generale pentru proiectie
– Filmele se citesc împreună.
– RgCP PA este privită ca și cum pacientul ar sta în fața voastră, cu dreapta lui în stânga voastră –
inima este în dreapta imaginii pe care o vedeți.
– Pe filmul lateral, pacientul privește de obicei înspre stânga (profil stâng).
– Filmele vechi sunt indispensabile.
– dacă aveți filme vechi, acestea se așează pe negatoscop lângă cel nou

Includerea: O RgCP trebuie sa includă intreaga cușcă toracică.


– Prima coastă și apex-urile pulmonare?
– Unghiurile costo-frenice?
– Marginile laterale ale coastelor?

Apariția unor structuri anatomice poate fi artifactuală.


Tehnică radiografică incorectă

Apariția neclară a unei structuri din cauza unei tehnici incorecte (rotație, inspir incomplet, penetrație
incorectă, haine sau alte obiecte care nu sunt indepartate).
Factori care tin de pacient

Necooperare în ceea ce privește poziționarea sau mișcarea, obezitatea, diverse structuri anatomice
Prezența unor obiecte externe/interne artificiale
•Artefactele pot fi inevitabile, dar unele dintre ele pot duce la interpretarea greșită a unei imagini.
•Totusi, artefactul este acceptabil în cazul în care problema clinică poate fi rezolvată

87. Analiza imaginii radiologice pentru: rotație, inspir, penetrație – criterii de


pozitivitate
•Rotație – apofizele spinoase pe linia mdiană a sternului, la jumătatea
distanței dintre capetele mediale ale claviculelor?
•Inspir – 6 coaste care intersectează diafragmul pe linia medio-
claviculară?
•Penetrație – coloana vertebrală vizibilă în interiorul opacității
cardiace?

88. Termenii folosiți pentru descrierea imaginii radiologice


Scopul examenului radiologic constă nu numai în identificarea și caracterizarea anomaliilor, ci și în
comunicarea acestor anomalii.Termenii nu trebuie sa fie confuzi!
Trebuie acordată atenție unor factori ca localizare, dimensiune, formă, densitate.
•Țesutul/organul afectat – plămâni, inimă, aortă, oase etc.
•Dimensiune – mare/mic/variat; se preferă descrierea în centimetri
•Partea – dreaptă/stângă; unilateral/bilateral
•Număr – unic/multiplu
•Distribuție – focală/diseminată
•Poziție – anterior/posterior/zone pulmonare etc.
•Formă – rotundă/semilunară etc.
•Contur – neted/neregulat/cu spiculi
•Pattern – nodular/reticular
•Densitate – aer/grăsime/țesut moale/calciu/metal

89. Zonele care necesită reexaminare pe RgCP


Evaluareaproiectiei : se cauta semnele care semnalizeaza stanga/dreapta (bula de gaz a stomacului,
semnalizarea tehnicienilor cu markeri).
Formula mnemotehnica - ABCDE
•Apexuri pulmonare – pneumotorax?
•Oase (Bones)/țesuturi moi – fracturi/densificari?
•Cordul – consolidări/mase?
•Diafragm – pneumoperitoneu?
•Colțurile (edges) imaginii – descoperiri neașteptate?
90. Enumerați semnele radiologice cu creșterea și cu scăderea densității
Asimetriile densității pulmonare apar fie ca zone mai albe (creșterea densității), fie ca zone mai închise la
culoare (scăderea densității).

– Cu creșterea densității – cele mai frecvente

Consolidare/Condensarea:Zonă de țesut pulmonar (în mod normal compresibil) care a fost umplut cu
lichid, caz în care zona devine indurată (consolidată).
– Exudat/puroi – pneumonie
– Transudat – edem pulmonar
– Sânge – hemoragie pulmonara
– Celule – cancer
Consolidarea trebuie sa fie prezentă pentru diagnosticul de pneumonie: semnele de pneumonie lobară
sunt caracteristice și sunt denumite clinic sindrom de condensare pumonară.
Atelectazie = Retracția (colapsul) alveolelor pulmonare
Este de 2 feluri:
 Atelectazie de resorbție (absorbție) (atelectazia obstructivă)
– Blocarea unei bronhii → absorbția aerului din plămâni
 Atelectazie de relaxare (retracție) (atelectazia non-obstructivă)
– Pneumotorax, pleurezie → detașarea plămânului de peretele toracic → retracția plămânului
prin forța de retracție elastică

Nodul/masă :
Solitar(ă) =o opacitate discretă, bine delimitată, rotundă, cu diametrul ≤3cm , complet înconjurată de
parenchim pulmonar, care nu atinge hilul și nu este asociată cu adenopatie, atelectazie sau epanșament
pleural
Multipli(e) = leziuni >3cm care pana la proba contrarie sunt considerate formatiuni maligne.
Afectare interstițială
– Cu scăderea densității

Difuz – emfizem
Localizat – chist, cavernă.

91. Bronhograma aerică


Aspect tubular al căilor aeriene facut vizibil de umplerea alveolelor inconjuratoare prin transudat
(edem) sau exudat (inflamație).
Cinci cauze
–Consolidare pulmonară
–Atelectazie non-obstructivă
–Pneumopatie interstițiala severă
–Neoplasm
–Expir normal

92. Indicațiile testelor funcționale pulmonare

93. Contraindicațiile testelor funcționale pulmonare:


– Chirugie oftalmologică recentă
– Anevrisme toracice, abdominale sau cerebrale
– Hemoptizie activă
– Pneumotorax
– Angor pectoris instabil/IM recent (<30zile)
94. Testul Sabrasez de ,,ținere” a respirației:
Cereți pacientului să inspire profund și să-și țină respirația cât mai mult posibil
>25 sec – rezerva cardio-pulmonară normală
15-25 sec – rezerva cardio-pulmonară limitată
<15 sec – rezerva cardio-pulmonară mult scăzută (contraindicație pt. chirurgie toracică)
Sau
25-30 sec CV ≈ 3500mL
20-25 sec CV ≈ 3000mL
15-20 sec CV ≈ 2500mL
10-15 sec CV ≈ 2000mL
5-10 sec CV ≈ 1500mL

95. Testul Schneider de stingere a lumânarilor:


– măsoară capacitatea respiratorie maximă
Efectuare - Cereți pacientului să stingă o lumânare de la o distanță de 15cm cu
•Gura larg deschisă
•Mandibula în repaus/susținută
•Fără strângerea buzelor
•Fără mișcarea capului
•Fără curenți de aer în cameră
•Gura și flacăra lumânării la același nivel

Interpretare
•Pacientul nu poate sa stingă lumânarea
•MBC <60 L/min
•FEV1 <1.6 L
•Pacientul este capabil sa stingă lumănarea
•MBC >60 L/min
•FEV1 >1.6 L
96. Spirometria: definiție, utilitate, criterii de validitate:
“Spirometry is a medical test that measures the volume of air an individual inhales or exhales as a function
of time.”
•Spirometrul măsoară volume;
•Spirometrul nu poate măsura – FRC, RV, TLC;
Procedura:
1.De obicei, testul incepe printr-o perioada de respiratie linistita (volum curent).
2.Apoi, i se cere pacientului sa inspire cat mai adanc poate , apoi sa expire cat mai repede și puternic poate
(expiratie fortata), peferabil cel putin 6 secunde.
3.De obicei, urmeaza un inspir rapid și maxim (inspiratie fortata).

Criterii de validitate
Start adecvat al testului – fara nicio ezitare
•Fara tuse / inchidere glotica
•Fara flux variabil
•Fara terminare precoce (<6 sec)
•Fara pierdere de aer
•Reproductibilitate – testul este fara variabilitate excesiva
Cele 2 cele mai mari valori ale FVC și cele 2 cele mai mari valori ale FEV1 nu trebuie sa varieze cu peste
0.2L.
97. Obiectivele și parametrii spirometriei forțate
Include masurarea:
Mecanicii pulmonare
– pentru a evalua capacitatea plamanului de a deplasa mari volume de aer rapid prin caile respiratorii
si
– Identificarea obstructiei cailor aeriene
Parametrii spirometriei fortate
•Capacitatea vitala fortata (FVC)
•Volumul expirator fortat (maxim) (FEV)
•Fluxuri expiratorii fortate (maxime) (FEF)
•Fluxuri inspiratorii fortate (maxime) (FIF)
•Ventilatia voluntara maxima (MVV)
98. Volume pulmonare statice
Volumul curent (TV) –500 mL
–Volumul de aer inspirat/expirat în cursul fiecarei respiratii curente (de repaus)
Volumul inspirator de rezerva (IRV) –3000 mL
–Volumul maxim de aer care poate fi inspirat din pozitia tele-inspiratorie a respiratiei curente
Volumul expirator de rezerva (ERV)–1500 Ml
–Volumul maxim de aer care poate fi expirat din pozitia tele-expiratorie a respiratiei curente
Volumul rezidual (RV)–1200 mL
–Volumul de aer restant în plamani dupa expirul maxim
–1200 mL
–Masurat indirect (FRC-ERV)
–Nu poate fi determinat prin spirometrie
99. Capacități pulmonare statice
Capacitatea inspiratorie (IC)–3400-3800 mL
–IRV + TV
–Volumul maxim de aer care poate fi inspirat din pozitia tele-expiratorie a respiratiei curente
Capacitatea expiratorie –2000 mL
–TV + ERV
–Volumul maxim de aer care poate fi expirat din pozitia tele-inspiratorie a respiratiei curente.
Capacitatea reziduala funcționala (FRC)- Nu poate fi determinată prin spirometrie
–RV + ERV
sau
–Volumul de aer din plamani în pozitia tele-expiratorie a respiratiei curente
–2500mL
–Scade in
•Pozitie supina
•Obezitate
•Inducerea anesteziei – 16-20%
100. Funcțiile capacității reziduale funcționale
•Depozit de oxigen
•Tampon pentru mentinerea unei PO2 arteriale stabile
•Inflatia partiala ajuta la prevenirea atelectaziei
•Minimizeaza munca respiratiei
101. Testul expirator forțat (ca semn auscultator, metode, interpretare)
–Dupa un inspir profund, pacientul efectuează un expir maxim forțat.
–Asezati stetoscopul pe trahee
•TEF normal 3-5 sec
•Boală pulmonară obstructivă >6sec
•Boală pulmonară restrictivă <3 sec
102. Teste funcționale la patul bolnavului: enumerare și scurtă descriere
1.Frecvența respiratorie: Parametru indispensabil în protocoalele de intubație & extubație
2.Testul Sabrasez de “ținere” a respirației: evalueaza rezerva cardio-pulmonară sau CV
3.Testul Schneider de stingere a lumânărilor + testul lumanarii modificat: masoara capacitatea
respiratorie maxima;
4.Testul tusei: tuse inadecvata in cazul in care FVC<30 ml/kg, FEV1<15ml/kg, PEFR<200L/min;
5.Timpul expirator forțat
•TEF normal 3-5 sec
•Boală pulmonară obstructivă >6sec
•Boală pulmonară restrictivă <3 sec
6.Peak flow-meter-ul Wright:
masoara PEFR (peak expiratory flow rate). <200 L/min => eficiență inadecvată a tusei
7.Microspirometrie
8.Pulsoximetrie
9.Gazele sanguine arteriale
103. Capacitatea vitală forțată
= Volumul maxim de aer care poate fi expirat cat mai rapid posibil dupa un inspir maxim
–Inspir maxim (pana la TLC), urmat expir abrupt pana la RV.
–Indirect, reflecta proprietatile de rezistenta la flux ale cailor aeriene.
Interpretare în % din FVC prezisa
–80-120% normal
–70-79% reducere usoara
–50-69% reducere moderata
–<50% reducere severa
104. Măsurări derivate din curba FVC
Volumul expirator maxim pe secunda (FEV1)
–volumul expirat în prima secunda a manevrei FVC
–Masoara severitatea generala a ovstructiei cailor aeriene
–Normal = 3000-4500L
 FEV1 – scade atat în bolile obstructive cat și în cele restrictive (in cazul în care capacitatea
vitala a pacientului este mai mica decat FEV1 prezisa).
 FEV1/FVC – redusa în bolile obstructive
Interpretare în % din valorile prezise
–>75% normal
–60-75% obstructie usoara
–50-59% obstructie moderata
–<49% obstructie severa
105. Curbele flux-volum-descriere și importanța clinică:
Curbele flux-volum (sau pneumotahograme) sunt fluxuri raportate la volume dinamice masurate.
Se obtin atunci cand un expir maxim fortat pornind de la TLC la RV este urmata de un inspir maxim fortat
inapoi la TLC.
–Masoara debitele inspiratorii și expiratorii fortate.
–Augmenteaza rezultatele spirometriei.
–Avantajul principal a anselor flux-volum vs. spirometria standard – identifica locatia anatomica
probabila a fluxului obstruat.
106. Debitul expirator maxim-importanța, caracteristici, valori normale
=Debitul maxim în cursul efectuarii unei manevre pentru FVC – apare în prima 0.1sec.
•Dupa un inspir maxim, pacientul expira cat mai fortat și cat mai repede poate și se masoara debitul maxim
al aerului.
•Ofera informatii despre functia pulmonara, reflectand functia cailor aeriene mari.
•Este dependenta de efort, dar inalt reproductibila.
•PEF la adultii normali variaza în functie de varsta și inaltime.
•Normal
–450-700 L/min la barbati
–300-500 L/min la femei
–Semnificatie clinica – valorile de <200 L/min fac tusea ineficienta, de unde posibilitatea unor
complicatii post-operatorii.
107. Ventilația voluntară maximă-valori normale și importanța clinică
•Masoara viteza și eficienta umplerii și a golirii plamanilor în timpul efortului respirator maxim
•Volumul maxim de aer care poate fi respirat în 1 minu de effort voluntar maxim
•Reflecta ventilatia de varf la necesitatile fiziologice
•Normal: 150-175 L/min
•<80%- alterare severa
MVV scade semnificativ la pacientii cu
–Emfizem pulmonar
–Obstructie a cailor aeriene
–Scaderea fortei muschilor respiratori
108. Toracocenteza-locație și indicații
= este o procedura invaziva de indepartare a lichidului sau aerului din spatiul pleural in scopuri diagnostice
sau terapeutice.Pentru aceasta, se introduce un ac in spatiul pleural, in general dupa administrarea unei
anestezii generale/locale.
Scop diagnostic - epansament pleural de etiologie neprecizata.
–In peste 90% din cazuri, analiza lichidului pleural duce la obtinerea unor informatii utile clinic.
–Cele mai frecvente cauze ale epansamentului pleural sunt cancerul, insuficienta cardiaca
congestiva, pneumonia si , recent, interventiile chirurgicale. In tarile cu o prevalenta inalta a
tuberculozei, si aceasta este o cauza frecventa a epansamentului pleural.
Scop terapeutic - in cazul in care exista cantitati mari de fluid,
–Pentru indepartarea acestui lichid si ameliorarea functiei pulmonare si a confortului pacientului.
–Atunci cand status-ul cardio-pulmonar este compromis (de ex. atunci cand lichidul pleural sau
aerul de la acest nivel are repercusiuni asupra functiei inimii/plamanilor), se recomanda
toracostomie cu drenaj – plasarea unui tub de dren in cavitatea pleurala.
109. Contraindicațiile și complicațiile toracocentezei
Contraindicatii relative
–Pacient necooperant
–Tulburari de coagulare necorectabile
–Leziuni buloase la nivelul punctiei (bule de emfizem)
–Un unic plaman functional
In niciun caz, aspiratul nu trebuie sa depaseasca 1L – risc de aparitie a edemului pulmonar.
110. Evaluarea epanșamentului pleural folosind criteriile Light
Transudatul este produs prin filtrare simpla (fara leziuni ale capilarelor), in timp ce exudatul este
“lichid inflamator”.
Epansamentele pleurale transudative se definesc ca epansamente determinate de factori sistemici
care altereaza fortele Starling.Componentele fortelor Starling (presiunile hidrostatice, presiunile oncotice si
permeabilitatea), determinante pantru apariția transudatelor sunt modificate in numeroase afectiuni, de
exemplu in insuficienta ventriculara stanga, insuficienta renala, ciroza hepatica.
Epansamentele pleurale exudative sunt determinate de factori locali care influenteaza formarea si
absorbtia lichidului pleural (de exemplu pneumonie bacteriana, cancer, embolism pulmonar, infectii virale)
si care implica lezarea capilarelor pleurale.
Diagnosticul acurat al cauzei epansamentului (transudat vs. exudat) se realizeaza pe baza unei
comparatii intre datele chimice ale lichidului pleural cu cele ale sangelui, folosind criteriile Light.
In cazul in care exista dubii asupra sensibilitatii si specificitatii criteriilor Lght se masoara albumina
din sange si lichidul pleural.Daca diferenta dintre albumina plasmatica si albumina din lichidul pleural este
>1.2g/dL, aceasta sugereaza un epansament transudativ.

111. Aspectul macroscopic al epanșamentului pleural și cauza posibilă


•Limpede, sero-citrin:
–Daca este transudat, nu mai este nevoie de o alta analiza.
–Daca este exudat, este nevoie de alte analize pentru determinarea cauzei (citologie, cultura,
biopsie).
•Tulbure, purulent, turbid: infectie, empiem, pancreatita, cancer.
•Roz-rosu/sangeriu: accident punctional, cancer, infarct pulmonar, infarct intestinal, pancreatita,
traumatism toracic.
•Verde-albicios, turbid: artrita reumatoida cu epansament pleural.
•Verde-brun: afectiune biliara, perforatie intestinala cu ascita.
•Alb-laptos sau galben-sangeriu: epansament chilos.
•Laptos-verzui, luciu metalic: epansament pseudo-chilos.
•Vascos (hemoragic sau limpede): mezoteliom.
•Pasta de ansoa (“sos de ciocolata”): ruptura de abces hepatic amoebian.
112. Examenul sputei-indicații
In boli precum: traheobronsite, pneumonii, bronhopneumonii, examenul sputei reprezinta o metoda
importanta de diagnostic etiologic.
113. Prelevarea sputei
Prelevarea sputei se face din expectoratia de dimineata, cand pacientul isi face de obicei „toaleta
bronhiilor”. In prealabil bolnavul va efectua o gargara cu ser fiziologic steril sau cu apa fiarta si racita daca
recoltarea se face la domiciliu. Pacientul trebuie instruit asupra obtinerii sputei propriu-zise si nu a salivei
sau a secretiilor nazale.
Pacientii care nu pot expectora trebuie asistati de cadre medicale (prin folosirea de aerosoli care
induc sputa sau pozitii drenante).
Produsul este prelevat intr-un recipient steril, cu gat larg, prevazut cu capac. In infectiile acute este
suficient sa se recolteze un volum de 1-2 ml secretie purulenta.
Specimenul este transportat la laborator imediat pentru a fi examinat in interval de cel mult o ora
de la prelevare.Refrigerarea la 4ºC previne multiplicarea contaminantilor, conserva unii patogeni, dar
scade pana la anulare izolarea altora, de exemplu, Haemophilus influenzae.
114. Aspectul macroscopic al sputei și cauze posibile

115. Bronhoscopia-definiție și indicațiile bronhoscopiei rigide


= Bronhoscopia este o tehnica endoscopică de vizualizare interiorul cailor respiratorii si in scopuri de
diagnostic si terapeutice.
Utilizari
–Extragerea de obiecte străine.
•Preferata pentru recuperarea corpilor străini aspirati , deoarece permite protejarea cailor aeriene
și controlul corpului străin în timpul recuperării.
–Hemoptizia masivă (definita ca pierderea de> 600 ml de sânge în 24 de ore) – mare urgență medicală.
•Lumenul mare permite abordari terapeutice, cum ar fi electrocauterizarea, care ajuta la controlul
sângerării.

116. Bronhoscopia flexibilă-indicații


Diagnostic
–Vizualizarea anomaliilor cailor respiratorii
–Prelevarea de probe tisulare. Probele pot fi luate din interiorul plamani prin biopsie, lavaj bronhoalveolar,
sau periaj endobronsic.
–Evaluarea hemoptiziei .
Terapeutic
–Eliminarea secretiilor, sângelui, sau obiecte străine depuse în căile respiratorii
–Rezectia laser a stricturilor maligne/benigne trahelae/bronsice.
–Insertia unui stent pentru inlaturarea unei compresii extrinseci a lumenului traheobronsic.
–Intubație traheală a pacienților cu cai aeriene dificile se realizeaza de multe ori cu ajutorul unui
bronhoscop flexibil.
117. Lavajul bronhoalveolar-indicații
Indicatii
–infectii la persoanele imunodeprimate
–pneumonii la pacientii vetilati
–unele tipuri de cancer pulmonar, si
–pneumopatii interstițiale.
Se poate folosi si in scop terapeutic
–Whole lung lavage (WLL; informal “spalare pulmonara”) – tratament pentru proteinoza alveolara.
118. Sindroame respiratorii infecțioase- definiție si clasificare
= Sindroame nespecifice produse de inflamații infecţioase acute ale tractului respirator.
1.Infecții acute ale căilor respiratorii superioare
– Rinita (coriza, guturaiul comun) – inflamația mucoasei nazale
– Sinuzita – inflamația sinusurilor paranazale
– Faringita (amigdalita) – inflamația faringelui, uvulei și a amigdalelor
– Laringita – inflamația laringelui
– Epiglotita – inflamația porțiunii sup. a laringelui și a ariei supraglotice
2.Infecții acute ale căilor respiratorii inferioare
– Traheita – inflamația traheei
– Bronșita – inflamația bronhiilor
– Bronșiolita acută – inflamația bronhiilor mici
3.Infecții ale parenchimului pulmonar (pneumonii)
– Pneumonia acută – inflamația parenchimului pulmonar
4.Răceala comună (rino-faringita)/Gripa
119. IACRS: tabloul clinic și abordare terapeutică
Simptome şi semne locale:
Rinita – inflamația mucoasei nazale
–Senzaţie de nas înfundat (determinată de inflamaţia mucoasei nazale şi sinusale)
–Strănut
–Hipersecreţie nazală (rinoree, coriză, guturai) - iniţial apoasă, ulterior purulentă (prin suprainfecţie
bacteriană)
–Anosmie (lipsa mirosului)
–Epistaxis minor prin ruperea capilarelor nazale
Sinuzita – inflamația sinusurilor paranazale
–Cefalee sau durere facială (în cazul afectării sinusurilor paranazale).
–Cacosmie (perceperea unor mirosuri neplăcute datorită secreţiilor nazale purulente)
–Halitoza (miros neplăcut al aerului expirat)
Faringita (amigdalita)– inflamația faringelui, uvulei și a amigdalelor
–Odinofagie – durere la deglutiție
Laringita – inflamația laringelui
–Disfonie – răgușeală
Epiglotita – inflamația porțiunii superioare a laringelui și a ariei supraglotice
–Disfonie
–Stridor
Anumite virusuri
–congestie conjunctivală ± lăcrimare abundentă;
Abordare medicală
–Trebuie apreciat dacă
•Există afectare sistemică (febra este rară)
•Exista prindere a CRI.
–Dacă există doar IACRS, cele mai multe sunt virale, deci auto-limitante și nu necesită antibiotic.
–La adulţii tineri, IACRS este frecvent cauzată de Mycoplasma pneumonie şi se poate manifesta prin tuse
neproductivă / slab productivă cu persistenţă de 4-6 săptămâni.
–Tratament simptomatic
–În caz de infecție bacteriana – antibiotic
–Cazuri particulare
•Infecțiile cu Streptococcus pyogenes
•Gripa
120. Rezultatele examenului fizic în IACRS
Examenul fizic - Semnele fizice polimorfe, inconstante şi nespecifice
–Inspecţie:
Eritem şi ragade perinazale
Rinoree
Congestie conjunctivală şi lăcrimare
Voce nazonată sau disfonie
Eritem şi exudate faringiene
–Palpare:
Punctele sinusale dureroase la presiune
Adenopatie cervicală uni- / bilaterală sau chiar generalizată (infecţia cu virus Epstein-Barr din
mononuleoză, cu virus citomegalic sau HIV)
Punctele sinusale dureroase - frontale, etmoidal şi maxilare
–Percuţie:
Nu aduce date utile diagnosticului.
–Auscultaţia:
•Tahicardie proporţională cu febra
121. IACRS-etiologie
Virală: majoritatea cazurilor.
–Virusurile implicate cel mai frecvent sunt rhinovirus, virusurile paragripale, coronavirus, adenovirus,
virusul respirator sinciţial, coxsackievirus şi virusurile gripale.
Bacteriană: mai rară, de obicei grefată pe o infecţie virală.
–Cele mai întâlnite tulpini sunt streptococul beta hemolitic grup A (SBHA), Corynebacterium diphtheriae,
Neisseria gonorrhoeae, Chlamydia pneumoniae, Mycoplasma pneumoniae, Streptococcus pneumoniae,
Haemophilus influenzae, Moraxella catarrhalis şi Bordetella pertussis.
122. Bronșita acută- definiție, etiologie și tabloul clinic
Etiologic
– infecție respiratorie acută, de obicei de origine virală (90%) : cORONAVIRUS, Rhinovirus,
Adenovirus, Virusul sincitial respirator, virusul gripal, virusii paragripali.
- Bacteriana si fungica : Mycoplasma pneumoniae, Chlamydophila (fosta Chlamydia) pneumoniae.
AP – inflamație de scurtă durată a bronhiilor de dimensiuni mari și medii
Clinic-paraclinic
–tuse, care evoluează cu/fără expectorație cu durată de până la 3 săptămâni.
–RgCP normală
Caracteristici generale
–Boală de obicei (90%) virală (deci, nu necesită antibiotic)
–Boală acută cu durata de 3-21 zile (dar tusea poate persista până la 6 săptămâni)
–Este deosebită de bronșita cronică, boală cronică nevindecabilă.
Debut
–Debutează adesea ca o IACRS, la care se adaugă tuse, de obicei productivă.
Perioada de stare
–Se caracterizează prin simptome generale și locale (predominent traheo-bronşice)
Simptome generale: similare celor din perioada de debut ale IACRS
–Maleză – stare generală de disconfort
–Astenie, adinamie, fatigabilitate
–Mialgii, poliartralgii,, anorexie
–Febră / subfebrilitate, frisoane
–Transpiraţii profuze
–Somnolenţă / stări confuzionale (bătrâni).
Simptome locale:
–Tuse iniţial uscată, ulterior productivă, cu spută sero-mucoasă / muco-purulentă (în cazul suprainfecţiei
bacteriene).
–Durere retrosternală cu caracter de arsură permanentă
–Pot apare şi spute hemoptoice prin leziuni ale mucoasei bronşice provocate de agentul viral, sau
secundare eforturilor de tuse.
–Dispnee
»expiratorie, uneori cu sibilante
»inspiratorie (în cazul afectării laringiene).
123. Rezultatele examenului fizic în bronșita acută
Semne similare cu cele din IACRS, la care se adauga semne bronșice
Inspecţie
•Eritem şi ragade perinazale
•Rinoree
•Congestie conjunctivală şi lăcrimare
•Voce nazonată sau disfonie, wheezing
•Eritem şi exudate faringiene
Palpare:
•Punctele sinusale dureroase la presiune
Percuţia
•nu aduce date utile diagnosticului.
Ascultaţia:
•Tahicardie proporţională cu febra
•Raluri bronşice uscate (sibilante şi ronflante)
124. Pneumonia-definiție, etiologie, aspecte clinice și paraclinice ( modificări
radiologice ) - generalități
Definitie= infecţii acute de obicei bacteriene (dar și virale, fungice, parazitare) ale parenchimului pulmonar
(alveole).
Clinic-paraclinic
–Manifestări clinice diverse:
•Febră >380C
•Tahipnee >24/min
•Tahicardie >100/min; cele mai importante sunt:
•Sunete respiratorii supraadăugate focale
•SaO2<88%
–RgCP: opacifieri – consolidare și/sau infiltrat interstițial.
În mod ideal, pacienții internați pentru pneumonie trebuie să primească prima doză de antibiotic
încă de la camera de gardă.
125. Pneumonia-administrarea antibioticelor și scorul de severitate ( CURB-65
- pneumonia de comunitate)
În mod ideal, pacienții internați pentru pneumonie trebuie să primească prima doză de antibiotic
încă de la camera de gardă.Înainte de apariția antibioticelor, pneumonia era a 3-a cauză de deces.
Pneumonia dobândită în comunitate (PdC)
–Pneumonie instalată în afara mediului spitalicesc, la persoane anterior sănătoase sau cu afecţiuni
respiratorii cronice stabilizate.
–Cele mai numeroase (>90%) și cu prognosticul cel mai bun.
–Un subset: “walking pneumonia”.
Scorul CURB-65 – câte un punct pentru fiecare din următoarele:
– Confuzie: dezorientare personală, temporală, spațială
– Uree serică >40mg/dL
– Frecvența Respiratorie: >30/minut
– PA (Blood pressure) sistolică: <90mmHg
– >65 ani
126. Pneumonia clasică-debut și evoluție
Debut
–Pneumonia clasică (pneumococică – determinată de Streptococcus pneumoniae) apare, de obicei, la copiii
mari și la adulți.
–Poate fi precedată de o boală virală urmată de un debut acut cu febră înaltă – adesea cu un unic frison
intens (frison “solemn”), durere pleuritică (junghi toracic), tuse productivă, dispnee, tahipnee, transpirații
profuze, astenie, stare generală de rău.
–Pacienții “arată rău”, sunt anxioși.
–Primele 24 ore
•Scăderea amplitudinii respiratorii pe partea afectată;
•Notă percuțională submată.
–Zilele 7-14 (faza de rezoluție)
•Cele mai multe semne dispar la sfârșitul celei de a 2-a săptămâni de boală.
•Atunci când incepe rezoluția, apar numeroase cracmente groase, care indică lichefierea exudatelor
alveolare.
–Zilele 2-7 – semnele de consolidare (condensare) → sindromul clasic de consolidare (pneumonia francă
lobară):
•I: scăderea amplitudinii respiratorii pe partea afectată.
•P: vibrații vocale exagerate (singura patologie pulmonară în care această manevră este utilă).
•P: nota percuțională submată.
•A:
•Sunet bronșic (suflu tubar);
•Sunete respiratorii supraadăugate (crepitante tele-inspiratorii fine);
•Frecătură pleurală (40%);
•Pectorilocvie afonă, egofonie, bronhofonie.

127. Pneumonia clasică-manifestări clinice generale și particulare


Manifestări generale și respiratorii
–Hipertermie (febra >38°C) sau hipotermie (<35°C);
–Tahipnee (>20 respirații/min);
–Utilizarea mușchilor respiratori accesori;
–Tahicardie (>100bătăi/min) sau bradicardie (<60bătăi/min);
–Cianoză centrală;
–Alterarea status-ului mental.

La pacienții vârstnici
–Febra este mai rară (de obicei, creșterea temperaturii este ușoară sau absentă).
–Frisoanele, transpirațiile, cefaleea, mialgiile – mai rare.
–Semnul cel mai important este tahipneea.
–Confuzia este mai frecventă.

Cea mai frecventă complicație a pneumoniei pneumococice este epanșamentul pleural (40% dintre
pacienți)
–Matitate percuțională la baze.
–Doar la 10% dintre pacienți, epnșamentul este suficient de voluminos pentru a permite toracenteza.
–Doar 10% intrunește criteriile de empiem.

128. Clasificarea pneumoniilor


1.Tipuri anatomo-patologice, clinice și radiologice
–Pneumonia lobară (pneumococică) = condensare pulmonara pleurezie (40%)
–Bronhopneumonia – bacteriană
–Pneumonia interstițială [atipică primară (PAP)] – viruși/mycoplasma
2.Etiologie
–Bacterii, mycoplasme, viruși, fungi
–Atenție: multe sunt precedate de IACRS
–Bacterii: Streptococcus pneumoniae este cea mai frecventa bacterie care dtermină, la nivel mondial,
pneumonie.
–Virale – mai frecvente la copii
3.Epidemiologic
–Pneumonia dobândită în comunitate (PdC)
–Pneumonia dobândită în spital (PdS)
–Pneumonia de aspiraţie (PA)
–Pneumonia la imunodeprimaţi (PID)
129. Clasificarea epidemiologică-argumentați importanța și descrieți succint
tipurile de pneumonii
1.Pneumonia de comunitate
–Pneumonie instalată în afara mediului spitalicesc, la persoane anterior sănătoase sau cu afecţiuni
respiratorii cronice stabilizate.
–Cele mai numeroase (>90%) și cu prognosticul cel mai bun.
–Un subset: “walking pneumonia”.
2.Pneumonia intraspitalicească (nosocomială)
–Pneumonia instalată la pacienţii internaţi de cel puţin 2 zile într-un serviciu spitalicesc.
–Până la 5% din pacienții internați pentru alte cauze dezvoltă pneumonie.
–Diferențe
•Pacienții spitalizați au multi factori de risc: ventilație mecanică, malnutriție, boli pulmonare și cardiace,
tulburări imune.
•Microrganismele intra-spitalicești sunt, de obicei, diferite de cele din comunitate
–Bacterii rezistente - MRSA, Pseudomonas, Enterobacter, Serratia.
•Tratamentul este, evident, diferit.
•Prognostic mai rău comparativ cu PdC
–Un subset: pneumonia asociată ventilației mecanice – apare la 48 ore după intubație și ventilație
mecanică.
3.Pneumonia de aspirație
–Pneumonia instalată după aspiraţia confirmată a conţinutului gastric sau în prezenţa factorilor de risc
pentru aspiraţie:
•Alterarea stării de conştienţă
•Tulburări de deglutiţie
•Obstrucţii intestinale
4.Pneumonia la imunodeprimați
–Pneumonia instalată la pacienţi cu una sau mai multe din următoarele caracteristici:
•leucocite <1000/mm3
•corticoterapie recentă
•tratamente citotoxice recente
•radioterapie recentă
•asplenie.

130. Pneumonia de comunitate-abordare terapeutică și complicatii


Etiologia este necunoscuta. Se trateaza cu antibiotic inainte de a cunoaste agentul cauzal.
Suspiciune PdC = Orice pacient cu simptome respiratorii recent instalate, mai ales dacă sunt însoţite de
febră şi semne auscultatorii pulmonare semnificative.
Asociere(i) care impun RgCP ≥1 din următoarele:
•Febra >380C
•Tahipnee >24/min
•Tahicardie >100/min
•Sunete supraadăugate focale
•SaO2<88%
Complicatii:

131. Importanța radiografiei cardiopulmonare în diagnosticul pneumoniei


– RgCP indică, de obicei, infecția înainte să apară semnele fizice ale pneumoniei atipice (pneumonie
ocultă).
– Leziunile sunt de tip infiltrat interstițial
– Infiltratele apar mai întâi în regiunea perihilară (unde încep bronhiile) și diseminează “in evantai”
către periferia câmpurilor pulmonare.
– Procesul interesează cel mai adesea lobii inferiori, dar poate afecta orice lob sau combinație de lobi.
Pneumonie francă lobară (a lobului mediu drept). Observati:
– Semnul siluetei
– Bronhogramă aerică
– Afectarea unui întreg lob
Pneumonie interstițială
– Întrucât inflamația este, de obicei, limitată la interstițiul pulmonar și septurile interlobulare, apare
aspectul de opacități reticulare sau reticulonodulare.
– Opacitățile apar în special în zonele pulmonare perihilare.
– Anomaliile radiologice sunt mai extinse decât ne-am aștepta după tabloul clinic.
Bronhopneumonie
– Opacități nodulare și reticulo-nodulare multiple cu tendința la confluare separate de parenchim
pulmonar normal.
– Distribuția este bilaterală și asimetrică, interesând predominant bazele pulmonare.

Infiltratele pulmonaredefinesc radiologic conceptul de pneumonie:


– Lobare – consolidarea unuia sau mai multor lobi
– Lobulare (bronhopneumonia) – consolidarea unuia sau mai multor lobuli secundari
– Interstiţiale
Radiografia standard furnizează informaţii despre
– severitatea bolii (afectarea multilobară bronhopneumonică)
– complicaţii (pleurezie, abces pulmonar)
– diagnosticul diferenţial al bolii.
Rezoluţia radiologică a infiltratelor este mai tardivă decât cea a simptomelor clinice (disociaţie radioclinică)
→ de aceea, în cazurile cu evoluţie favorabilă, necomplicată, controlul radiografic este inutil înainte de 4-6
săptămâni.
132. Bronhopneumonia – caracteristici
Bronhopneumonia (pneumonia bronhogenă, lobulară) este tot o consolidare, însă
 Debutează la nivelul căilor aeriene
 Infecția se propagă de-a lungul căilor aeriene
 Apar multiple focare de consolidare, afectând mai mulți lobuli pulmonari, la nivelul lobilor bazali, de
obicei bilateral.
 Bronșiolită acută (exudat supurativ intralumenal și inflamație parietala) + alveolită leucocitară (la
nivelul alveolelor adiacente)
 Leziunile au 2-2.5cm în diametru, sunt gri-gălbui, uscate, centrate pe o bronhiolă, slab delimitate,
cu tendință la confluență.
 Focarele inflamatorii sunt înconjurate de parenchim sănătos, aerat.
133. Pneumoniile atipice-etiologie și tablou clinic
 Sunt determinate de microorganisme atipice (altele decât Str. pneumoniae și St. aureus).
 În momentul descrierii pneumoniilor “atipice”, microorganisme ca Mycoplasma, Chlamydophila, și
Legionella nu erau încă recunoscute ca bacterii, ci erau considerate viruși (pneumonii “non-
bacteriene”).
 Între timp, multe asemenea microorganisme au fost clasificate ca bacterii, deși speciale (Mycoplasma
este un tip de bacterie fără perete celular, iar Chlamydiile sunt paraziți intracelulari).

Cei mai frecvenți agenți cauzali sunt bacterii (adesea cu viață intracelulară):
– Mycoplasma pneumoniae (cea mai frecventă cauză) – apare, de obicei, la tineri, are manifestări
ușoare, dar se poate asocia cu simptome neurologice și sistemice (de ex. rash).
– Chlamydophila pneumoniae – formă ușoară de pneumonie, cu simptomatologie minimă.
– Chlamydia psittaci - psitacoza.
– Coxiella burnetii – febra Q.
– Francisella tularensis - tularemia.
– Legionella pneumophila – formă severă de pneumonie, cu mortalitate relativ înaltă, cunoscută ca
legioneloza (sau boala legionarilor)
– Cauze virale - virusul respirator sincițial, virușii gripali A și B, adenovirusuri, etc.

134. Bronhopneumopatia cronică obstructivă-etiologie, patogenie,


fiziopatologie, alte particularități
Criteriul principal pentru diagnosticul BPCO este VEMS/CVF <70%.
Etiologie: agenți nocivi cum ar fi fumul de țigară, combustibilii solizi și agenți ocupaționali.
Anamneză de fumat – de departe cel mai semnificativ factor de risc
Anamneză de alte expuneri inhalatorii (poluarea aerului)
– Pulberi sau chimicale ocupaționale
– Expunere pasivă la fumat
Tendință genetică sau familiala la BPOC
– Deficitul de alfa-1-antitripsină (afectiune relativ rară, dar un factor de risc genetic subdiagnosticat al
BPOC).
Patogenie:
– Inflamație cronică a căilor aeriene mici (bronhiolele cu calibrul sub 2mm) şi a parenchimului
pulmonar (efector principal - PMN). Fibroză a căilor aeriene, dopuri lumenale. Creștere a rezistenței
căilor aeriene.
– Câteodată (adeseori) asociată cu hiperresponsivitatea căilor aeriene
Anatomo-patologic: două anomalii, de obicei coexistente
– Bronșiolita cronică obstructiva
– Distrucția septurilor interalveolare (emfizem)
Fiziopatologie
– Disfuncție ventilatorie obstructivă progresivă - VEMS/CVF <70%
– Care nu se poate normaliza spontan sau ca răspuns la tratament (bronhodilatatoare,
antiinflamatoare)
Clinic
– Boală prevenibilă, progresivă, ireversibilă.
– Disfuncție ventilatorie obstructivă progresivă predominant expiratorie (scade VEMS)
 Două tipuri de manifestări
– Manifestări pulmonare – în special dipnee
– Manifestări extra-pulmonare care contribuie la severitatea bolii (tulburări nutriționale, boală
cardio-vasculară, depresie, ostopenie, infecții cronice, “frailty”).

135. Emfizemul pulmonar-tablou clinic


Definit pe criterii anatomo-patologice:
•Distrugerea pereţilor alveolari şi dilatarea anormală și permanentă a spaţiilor aeriene distal de bronhiola
terminală.
Simptome
Dispnee: persistentă, progresivă, agravată la efort sau în timpul infecțiilor respiratorii.
•Întrebați întotdeauna pacienții în legatură cu gradul dispneei la anumite activități, cum ar fi urcatul
scărilor sau al pantelor ușoare, mersul alături de altcineva, îmbrăcat/dezbrăcat pentru a încerca să
apreciați severitatea simptomelor pacientului !!!
Tuse cronică, persistentă/intermitentă
Producție cronică de spută
•Inițial, disfuncția ventilatorie obstructivă din cadrul BPOC poate fi asimptomatică.
•Invers, tusea cronică și producția de spută pot preceda cu mulți ani limitarea fluxului de aer, desi nu toți
cei cu tuse productivă ajung să dezvolte BPOC.

136. BPOC- definiție și scara severității ( stadializarea clinică, NU Gold)


Limitarea fluxului aerian respirator (disfuncție ventilatorie obstructivă)
•Un grup de afecțiuni cu
– Mai multi factori etiologici;
– Mecanisme patogenice incomplet înțelese;
– Definiție anatomo-patologică nepractică;
– Simptomatologie clinică nespecifică.
•Anomalia de bază a pacienților cu BPOC este fiziopatologică: obstrucția ireversibilă a căilor aeriene.
137. BPOC – rezultatele examenului fizic
Inspecţia:
– Mărirea diametrului antero-posterior al toracelui (torace emfizematos, în butoi).
– Umplerea foselor supraclaviculare.
– Activarea musculaturii respiratorii accesorii.
– Prelungirea expirului, cu inversarea raportului inspir/expir.
– Tahipnee.
– Wheezing.
Percuţia:
– Hipersonoritatea ambelor arii pulmonare.
– Diminuarea matităţii cardiace.
Auscultaţia:
– Diminuarea sunetului pulmonar normal pe ambele arii pulmonare.
– Sunete supraadăugate diseminate bilateral, uneori modificate de tuse, alteori persistând şi după
încetarea respiraţiei (semnul cimpoiului).

138. BPOC - forma dispneică – scurte explicații anatomo- și fizio- patologice,


semne, simptome, complicații
Forma dispneică (pink puffer, pufăitorul rozaliu, BPOC tip A): element predominant – emfizemul
pulmonar:
AP: emfizem
Simptome
–Dispnee progresivă, tahipnee, ortopnee
–Tuse minima, ineficienta (sputa minima)
Semne
–Bătrân și slab, anxios
–Cianoză minimă/absentă (roz)
–Respirație “cu buzele strânse”
–Utilizarea mușchilor respiratori accesori și retracții intercostale
–I: Ampliații respiratorii reduse (torace fixat în inspir), semnul Hoover, torace în butoi, înălțimea laringiană
redusă
–P: Nota percuțională hipersonoră, coborârea matității hepatice
–A: murmur vezicular diminat (torace liniștit), fără sunete supraadăugate
–Timp expirator crescut
Complicații
–Pneumotorax
–Scădere ponderală

139. BPOC – forma tusigenă - scurte explicații anatomo- și fizio- patologice,


semne, simptome, complicații
Forma tusigenă (blue bloater, buhăitul albastru, BPOC tip B): element predominant – bronșita cronică.
AP: bronșită cronică

Simptome
–Tuse productivă mucoasă sau muco-purulentă recurentă
–Dispnee minimă (la debut)

Semne
–Supraponderal
–Coleric
–Cianoză (hipoxie + hipercapnie)
–Hipocratism digital
–I: mai puțin semnificative
–P: Nota percuțională hipersonoră, coborârea matității hepatice
–A: murmur vezicular normal, dar numeroase sunete supraadăugate (cracmente, sibilante și ronflante).

Complicații
–Poliglobulie (cresterea Hb) - consecutiva hipoxemiei
–HT pulmonară (vasoconstricție reactivă din cauza hipoxemiei)
–Insuficiență ventriculară dreaptă – edeme periferice

140. Protocolul evaluării funcționale a BPOC


1. Spirometria
Investigaţia standard, deci obligatorie pentru identificarea şi evaluarea severităţii BPOC.

Trebuie efectuată la:


– toți pacienții cu simptome cronice sugestive de BPOC;
– toți pacienții peste 40 de ani cu factori de risc pentru BPOC (în special fumat).

Trei parametri fundamentali


– FEV1 - evaluează patența căilor respiratorii.
– FVC - evaluează capacitatea pulmonară.
– Raportul FEV1/FVC: limita inferioară a normalului variază cu vârsta. Un raport redus (<70%) este
obligatoriu pentru diagnosticul BPOC.

Criteriile pentru
– Diagnostic
 Împreună cu evaluarea simptomelor, diagnosticul este confirmat atunci când raportul FEV1/FVC
<70% (variază în funcție de vârstă, sex, înălțime, etnie)(WHO Global Initiative for Chronic
Obstructive Lung Disease, GOLD, 2014)
– Severitate
 Împreună cu severitatea simptomelor, valorile obținute prin spirometrie clasifică severitatea
bolii.
Examenul spirometric va fi repetat cel puţin anual pentru evaluarea evoluţiei clinice şi a răspunsului la
tratament.
2. Testul de 6 minute de mers
3. Calcularea scorului BODE

143. Astmul bronșic-etiologie și patogenie


Etiologic:
Cauzele şi mecanismele creşterii reactivităţii bronşice la astmatic sunt
incomplet cunoscute.

Riscul de apariție a astmului bronșic este legat de o serie de factori etiologici:


–Factori genetici - studii recente au identificat cromozomii purtători ai genelor responsabile atât de
atopie, cât şi de hiperreactivitatea bronşică.
–Factori de mediu – cum ar fi expunerea la alergeni sau la poluanți aerieni
–Atunci când factorii genetici și cei de mediu coexistă, riscul este suplimentar augmentat.

Tradiţional, astmul este împărţit în 2 grupe etiologice:


–Astmul extrinsec (alergic, atopic) - datorat unei reacţii de hipersensibilitate de tip I la alergenii
pătrunşi pe cale inhalatorie.
–Astmul intrinsec - în care nu se poate pune în evidenţă hipersensibilitatea imediată la vreun
antigen specific.
Patogenic și anatomo-patologic
•Inflamație cronică => hiperreactivitate nespecifica(condiţionarea mastocitelor, eozinofilelor,
bazofilelor şi macrofagelor din mucoasa bronşică care elibereaza 2 categorii de mediatori: mediatori
ai hipersensibilitatii immediate si factori chemotactici)
•Remodelare a căilor aeriene => DVO persistenta

144. Tabloul clinic al crizei de astm


Simptomatologie - tetrada
1.Wheezing expirator – sunet “șuierător” apărut în cursul expirului (mai rar în ambele faze ale respirației)
2.Tuse neproductivă, în special nocturnă
3.Dispnee expiratorie recurentă
4.Constricție toracică recurentă

Simptomele pot fi ușoare, moderate sau severe – și pot varia de la un episod la altul; pot fi izolate sau
asociate:
–Apar sau se înrăutățesc noaptea, trezind pacientul
–Simptomele sunt episodice, câteodata sezoniere
–Apar sau se înrăutățesc în prezenta factorilor declanșatori.
Episodul acut se termină printr-un acces de tuse cu expectoraţie vâscoasă, redusă cantitativ, a cărei
eliminare uşurează bolnavul.
Eczemele, rinita alergică și anamneza familială de astm sau boală atopică se asociază frecvent cu astmul
bronsic, dar nu sunt indicatori anamnestici cheie.

145. Examenul fizic al crizei de astm

Inspecţia:
•Creşterea diametrului toracic antero-posterior, umeri înălțați
•Tahipnee cu inversarea raportului inspir/expir
•Utilizarea musculaturii respiratorii accesorii
•(posibil) cianoză

Percuţia
•Hipersonoritate; coborârea matității hepatice

Palparea
•Tahicardie ± puls paradoxal

Ascultaţia:
•Wheezing (inspirator și expirator)
•Diminuarea/dispariţia murmurului vezicular
•Sibilante şi ronflante („zgomot de porumbar”)
•Expir prelungit
•În perioadele intercritice examenul fizic al toracelui nu evidenţiază – în general – semne patologice.

Altele
•Creșterea secreției nazale, edeme ale mucoasei nazale sau polipi nazali
•Dermatite atopice, eczeme sau alte manifestari ale bolilor cutanate alergice.

146. Diagnosticul clinic în astmul bronșic și enumerarea investigațiilor


paraclinice
Diagnosticul clinic de astm bronşic poate fi formulat pe baza următoarelor criterii:
– Episoade nocturne de tuse, dispnee, wheezing și constricție toracică, simptome prezente izolat sau
în asociere.
– Simptome paroxistice şi rezolutive, deşi unul sau mai multe dintre ele poate/pot persista pe
perioade nedefinite.
– Istoric de hiperreactivitate bronşică manifestată prin exacerbarea simptomelor în momentul
expunerii la stimulii declanşatori ai bronhospasmului.
Investigatii paraclinice:
1. Spirometria trebuie aplicată la orice pacient cu vârsta >5ani
2. Debitmetria
3. Examenul sputei
4. Radiografia toracică în criză
5. Măsurarea gazelor sangvine:
6. Diagnosticul atopiei
7. Phadiatop-ul
8. Testele de provocare alergenică a bronhospasmului

147. Sindromul restrictiv – definiție, enumerare cauze


Grup heterogen de afecțiuni extrapulmonare si pulmonare a căror caracteristică comună este
incapacitatea de expansionare inspiratorie a plămânilor.
Aceasta determină:
–Scăderea volumelor și capacităților pulmonare → Ventilație inadecvată
–În cazul in care este vorba despre boli pulmonare:
•Îngroșarea membranei alveolare → reducerea transferului gazos (reducerea DLCO)
•Rigidizarea plămânilor → creșterea efortului respirator;
Numeroasele afecțiuni care pot determina DVR se pot grupa in 2 categorii, în funcție de structurile
anatomice afectate:
1. Boli extra-pulmonare
•Bolile neuro-musculare care afectează mușchii respiratori
•Bolile peretelui toracic
•Bolile pleurei
2. Boli ale parenchimului pulmonar
•Interstițiale
•Alveolare

148. Disfuncții ventilatorii restrictive pleurale – exemple, diagnostic*


– Pahipleurite
– Epanșamente pleurale
– Empiem cronic
– Asbestoza
Diagnostic:Rg, CT, Manometrie pleurală, Biopsie pleurala

149. Disfuncții ventilatorii restrictive alveolare – exemple, diagnostic


– Edem pulmonar
– Hemoragie pulmonara
Dioagnostic: Rg, CT, examen fizic;

150. Disfuncții ventilatorii restrictive interstițiale – exemple, diagnostic


– Pneumopatii interstițiale: Peste 300 de boli
Diagnostic: Rg, CT, examen fizic, ecografie CV (HTP)

151. Disfuncții ventilatorii restrictive neuromusculare – exemple, diagnostic


– Miasthenia gravis
– Scleroza laterală amiotrofică
– Miopatii
Diagnostic: Examen fizic , EMG, serologie.

152. Disfuncții ventilatorii restrictive toracice/extratoracice – exemple,


diagnostic
– Obezitate
– Cifoscolioza
– Ascita
Diagnostic: Examen fizic

153. Definitii pneumopatia interstitiala fibrozanta si sarcoidoza


Pneumopatia interstițială fibrozantă idiopatică
– Etio-patogenie necunoscută
– Anatomo-patologic: inflamație cronică, progresivă, fibrozantă a interstițiului pulmonar (pattern
radiologic de pneumonie interstițială uzuală).
– Clinic: boală care apare predominant la adultul tânăr, cu manifestări limitate la nivelul plămânilor
– Fiziopatologic: disfuncție ventilatorie restrictivă
Sarcoidoza
– Etio-patogenie necunoscută
– Anatomo-patologic: inflamație cronică, caracterizată prin apariția de granuloame non-cazeoase,
predominant la nivelul plămânilor și al ganglionilor limfatici intra-toracici.
– Clinic: boală multisistemică

154. Pneumopatii interstițiale – rezultatele examenului fizic

Datele sunt, de obicei, nespecifice:


Semne generale
– Hipocratism digital
– Cianoză
– Semne extrapulmonare ale afecțiunilor sistemice
Semne pulmonare:
– Tahipnee
– I: ampliații respiratorii reduse
– A: Cracmente tele-inspiratorii bibazilare “uscate” (velcro) – frecvente in pneumopatiile interstițiale
asociate cu inflamație, dar mai puțin probabil a apare în cele granulomatoase.
 Cracmentele pot fi prezente chiar în absența anomaliilor radiologice pe RgCP.
 Mai rar:
 Țipătul de pescăruș tele-inspirator (inspiratory squawks) – la pacienții cu bonșiolită.
 Examenul cardiac este normal, cu excepția stadiilor tardive de boală, când pot apare semne
de hipertensiune pulmonară și cord pulmonar.

155. Tipologii radiologice în pneumopatiile interstițiale


Pneumopatiile interstițiale sunt un grup de boli care afectează interstițiul pulmonar (țesutul și spațiul
care înconjoară spațiile aeriene ale plămânilor).
Tipologii radiologice:
Afectare/pattern reticular(ă) (în rețea) – cel mai frecvent, in general bibazilar

 Aspect de fibroză reticulară în mod caracteristic mai proeminent la nivelul bazelor pulmonare.
 Siluetele cardiace sunt mai puțin distincte – “shaggy”
 RgCP poate fi utilă pentru monitorizarea fibrozei pulmonare.

Afectare/pattern nodular(ă)
Afectarea/patternul reticulo-nodulara
În stadiile tardive, apar alte 2 pattern-uri, care indică un prognostic sever.
Creșterea densității pulmonare

– Plămân “în geam mat”


– Consolidare

Scăderea densității pulmonare


– Emfizem
– Boli chistice
– Plămân în “fagure de miere”

156. Sindromul bronșiectatic – definiție, enumerare cauze

Boală pulmonară obstructivă


•Etio-patogenic: combinația de secreție inflamatorie cronică și infecție bacteriană care determină
distrucția pereților bronșici și a țesutului suportiv înconjurător
•AP: dilatație permanentă, anormală a uneia sau mai multor ramuri ale arborelui bronşic.
•Clinic: infecții respiratorii recurente.
•Elementul central al diagnosticului este anatomo-patologic
•În 50% din cazuri, bronșiectaziile sunt idiopatice.
•Bronșiectaziile sunt clasificate ca boală pulmonară obstructivă, împreună cu BPOC și astmul bronșic.

157. Enumerați infecțiile necrotizante ce pot determina bronșiectazi


Infecţii necrotizante ale peretelui bronşic:
•Pneumonie cu Klebsiella pneumoniae, Staphylococus aureus
•Tuberculoza pulmonară
•Pneumonii cu virusuri gripale, paragripale, adenovirusuri, virusul rujeolic, etc.
•Aspergiloză pulmonară alergică

158. Enumerați cauzele obstructive ce pot determina bronșiectazii


Bronșiectazii obstructive - obstrucţia bronşică a căilor respiratorii proximale → acumularea secreţiilor în
amonte → suprainfecţie + necroză + dilatarea peretelui bronşic.
•Procese patologice intralumenale (corp străin, bronholitiază, tumoră benignă cu creştere lentă)
•Compresii extrinseci (adenopatie peribronşică voluminoasă, chisturi, tumori, etc.).
•Mucovoscidoza (fibroza chistică)
3.Retracţia parenchimului pulmonar cu deplasarea şi tracţiunea bronşiilor
•Fibroza pulmonară
•Rezecţii lobare.

159. Enumerați afecțiunile congenitale ce pot fi implicate în etiologia


bronșiectaziilor
Afecţiunile congenitale
•deficitul mecanismelor de apărare locală
– fibroza chistică (mucoviscidoza)
– deficitul de alfa-1 antitripsină
– diskinezia ciliară primitivă din sindromul Kartagener = bronşiectazie + sinuzită cronică +
dextrocardie/situs inversus + infertilitate
•malformaţii ale căilor respiratorii
– traheo- bronhomegalia din sindromul Mounier-Kuhn
•deficit imun sistemic
– hipogamaglobulinemia primitivă.
160. Explicați teoria cercului vicios în bronșiectazii

– Procesul debutează prin leziune inflamatorie exagerată, care determină distrucția pereților căilor
aeriene.
– Bronhia afectată își crește diametrul și devine incapabilă de eliminarea secrețiilor.
– Creșterea de volum a secrețiilor duce la înmulțirea bacteriilor intrapulmonare și la apariția unui cerc
vicios.

161. Bronșiectazii – anamneza și rezultatele examenului fizic


Anamneza
–Tuse productivă
–Spută purulentă masivă, uneori cu mires fetid – afecțiunea cu sputa cea mai voluminoasă.
–Anamneza de tuse productivă cu spută purulentă care se întinde pînă în copilărie este practic
diagnostică.
–Mai rar:
•Hemoptizie recurentă
•Febră și transpirații nocturne
•Anamneză de infecții respiratorii recurente
•Pierdere ponderală
Examen fizic
–Adesea neconcludent
Manifestări sistemice: febră, cașexie, sinuzită (70%)
Spută (nu uitați să vă uitați întotdeauna în flaconul pentru spută): purulentă, masivă, urât mirositoare,
câteodată cu striuri sanguinolente.
Auscultație: cracmente bilaterale, groase, proto-mezo-inspiratorii
Semne ale bronșiectaziilor severe:
– Cantități mari de spută asociate cu hemoptizie
– Hipocratism digital
– Cianoză
– Cracmente pe arii extinse
– Semne de obstrucție a căilor aeriene
– Semne de insuficiență respiratorie și cord pulmonar
– Semne ale amiloidozei secundare (edeme prin proteinurie, insuficiență cardiacă,
hepatosplenomegalie, sindrom de tunel carpian)

162. Clasificarea Reid a bronșiectaziilor cu scurte precizări pentru fiecare tip


În 1950, Reid a corelat modificările anatomo-patologice cu bronhografia și a descris trei tipuri de
bronșiectazii:
Cilindrice:
•Bronhii uniform dilatate (>2mm Ø <1cm).
•Se termină abrupt.
•Produse de obstruarea bronhiilor mici prin dopuri mucoase.
Varicoase:
•Bronhii dilatate cu diametru neregulat (asemănător unor vene varicoase).
•Terminațiile sunt bulboase, iar subdiviziunile bronșice sunt reduse.
Chistice (saculare):
•Forma cea mai severă.
•Subdiviziuni bronșice brusc reduse și bronhii dilatate care se termină în cavități bronșice pline cu
puroi.
•Toate cele trei tipuri pot fi prezente la același pacient.

163. Aspectele generale radiologice în bronșiectazii


RgCP
–Este rareori normală, dar modificările apărute sunt adesea nespecifice.
–Bronşiectaziile cilindrice pot da imagine:
•De ”şine de tramvai” - în bronșiectaziile cilindrice (dacă sunt surprinse în secţiune longitudinală)
•Inelară sau în “fagure de miere” – în bronșiectaziile varicoase/chistice.
–Datorită secreţiilor stagnante, lumenul bronşic apare de obicei mai radioopac decât parenchimul din
vecinătate.
- Bronșiectaziile pot exista chiar dacă RgCP este normală – mulți pacienți suspectați a avea bronșiectazii
necesită TC de înaltă rezoluție pentru confirmarea diagnosticului.

164. Complicațiile bronșiectaziilor


Complicații locale
–Pneumonia de însoţire.
–Hemoptizia care poate deveni voluminoasă (peste 600 ml), cu risc vital.
Complicaţii la distanta
–Caşexie.
–Anemia normocrom normocitară.
–Amiloidoza.

165. Sindromul atelectatic – definiție


Situație în care parenchimul pulmonar suferăun proces de închidere (colaps) prin pierdere a aerului
sau nu se poate expansiona complet.
Este o situatie patologică în care alveolele sunt deflate, fiind, în felul acesta, opusul
consolidării.Poate afecta o parte sau un întreg plămân si duce la reducerea sau absența schimburilor
gazoase.

166. Atelectazia obstructivă (resorbtivă ) – cauze, aspectul pleural


Atelectazia obstructivă (resorbtivă) – atelectazia cu bronhie închisă.
–Tipul cel mai frecvent și mai semnificativ clinic.
–Determinat de obstrucția competă a unei căi aeriene.
Cauze
– Corp străin
– Tumora endobronșică
– Dop mucos.
Obstrucția poate apare la nivelul bronhiilor mari/mici.
Patogenie: resorbția aerului din zona pulmonar neaerată → pleurele viscerală și parietală nu se despart →
apare tracțiune → structurile mobile ale toracelui sunt atrase înspre partea cu pierdere de volum.
Clinic= sindrom de condensare retractilă

167. Atelectazia obstructivă – anamneza


Anamneza:
Simptomele și semnele sunt determinate de:
– Rapiditatea de instalare a obstrucției bronșice
– Dimensiunea ariei pulmonare afectate
– Prezența/absența complicațiilor infecțioase.
1. Ocluzia bronșică rapidă cu colaps pulmonar masiv
– Durere pe partea afectată, dispnee brusc instalată, cianoză
– hTA, tahicardie, febră, șoc.
2. Atelectazia cu dezvoltare lentă
– Asimptomatică
– Simptomatologie minimă

168. Atelectazia obstructivă - rezultatele examenului fizic


Examen fizic – sindrom de condensare retractilă
– Trahee deplasată homolateral
– I: aplatisarea toracelui și expansiune toracică redusă homolateral
– P: vibrații vocale scăzute
– P: notă percuțională submată
– A: sunete respiratorii reduse/absente, sunete supraadăugate absente (calea aeriană este închisă).

169. Atelectazia obstructivă – aspect radiologic


RgCP
1. Creșterea densității (opacifiere) la nivelul ariei pulmonare atelectatice cu apariția semnului siluetei.
2. Pierdere de volum a ariei afectate.
– Deplasarea fisurilor către aria atelectatică a plămânului
– Deplasarea în sus a hemidiafragmului ipsilateral
– Diminuarea distanței dintre coaste
– +/- deplasarea mediastinului (în cazul în care atelectazia este substanțială)
3. Aglomerarea vaselor pulmonare și a bronhiilor în regiunea atelectatică.
4.+/- hiperinflație compensatorie (vicariantă) a plămânului neafectat.

170. Atelectazia non-obstructivă : cauze, tipuri, aspect pleural


1.Atelectazie “de relaxare” (atelectazie pasivă)
– Pierderea contactului dintre pleura viscerală și cea parietală;
– Apare în pleurezii, pneumotorax.
2.Atelectazia “de compresie”
– Orice afecțiune intratoracică dislocuitoare de spațiu poate comprima plămânul și scoate aerul din
alveole.
3.Atelectazia “de cicatrizare”
– Cicatrizarea (fibroza) reduce expansiunea pulmonară.
– Apare în pneumoniile interstițiale fibrozante, pneumopatia radică.
4.Atelectazie “adezivă”
– Pierderea surfactantului (care are rolul fiziologic de reducere a tensiunii superficiale) crește
tendința către colaps a acestor structuri.
– Apare în sindromul de detresă respiratorie acută
5.Atelectazia “dependentă” (gravitațională)
– Singura atelectazie care poate apare la persoanele sănătoase.

171. Atelectazia non-obstructivă – rezultatele examenului fizic


Examen fizic – sindrom de condensare (consolidare)
–I: trahee deplasată homolateral
–P: vibrații vocale crescute
–P: nota percuțională submată
–A: sunet respirator bronșic (suflu tubar), sunete supraadăugate absente – deosebire importantă față de
situația din pneumonia francă lobară

172. Sindromul de lob mijlociu ( sindromul Brock ) – definiție, scurte precizări


Definiție
•Colaps recurent al lobului mijlociu drept (sau al lingulei), cu/fără obstrucție a bronhiei, de obicei asociat
cu o bronșiectazie.

Patogenie: bronhia lobară a LMD are niște particularități importante:


•Lungă și subțire
•Are cel mai slab drenaj dintre toate bronhiile lobare (ceea ce favorizează retenția de mucus).
Clinica
•Apare, de obicei, la vârstnici, cu o predominanță pentru femei.
•Pacienți sunt, de obicei, asimptomatici, dar se pot identifica anamnestic tuse productiva cronica și
pneumonii drepte recurente.
•Important – de obicei, bronhia este patentă.

173. Atelectazia rotundă – definiție, cauze, semne radiologice specifice


Definiție
•Reprezintă țesut pulmonar atelectatic unit de pleura viscerală prin benzi fibrotice aderențiale.
Clinica
•Incidență mare la lucrătorii cu asbest (60-70% din cazuri).
Examenul radiologic
 Poate fi confundată cu tumora pulmonară/pleurală.
 Semnele radiografice caracteristice
– Îngroșare pleurală
– Semnul “cozii de cometă”.
 Teoria spune că o pleurită locală determină îngroșarea și contractarea pleurei viscerale. Plămânul
subiacent se contractă și dezvoltă o configurație rotundă. Vasele distorsionate atașate seamănă cu
o “coadă de cometă”.

174. Atelectazia în placă – cauze, mecanism, aspect radiologic


Este tipul cel mai frecvent vazut pe radiografiile toracice.
Cauze
•Hipoventilație, embolie pulmonară, IACRI,
•dar apare și la pacienți cu respirație superficială – fumători, vârstnici, chirugie abdominală.
Mecanism
•Ventilație regională inadecvată (posibil obstrucția unei bronhii mici)
•Anomalii în formarea surfactantului (determinate de hipoxie, ischemie, expuneri la diverse toxine).
Aspect radiologic
•Umbre lineare de densitate crescută la bazele pulmonare.
•De obicei orizontale, grosime de 1-3mm, lungime de cățiva centimetri.
Semnificație clinică – minimă.
175. Atelectazia post-chirurgicală – cauze, aspect radiologic
Definiție
•Complicație pulmonară frecventă la pacienții supuși unei intervenții toracice sau abdominale superioare.
Mecanism
•Anestezia generală și manipularea chirurgicală determină disfuncție diafragmatică și diminuarea activității
surfactantului.
•Inducerea anesteziei generale scade cu 7-11% volumul pulmonar total.
Semne radiografice
•Atelectazie bazală și cu distributie segmentală.

176. Complicațiile atelectaziei


•Pneumonia acută
•Bronșiectazia
•Hipoxemia și insuficiența respiratorie
•Septicemia
•Epanșament pleural și empiem

177. Sindromul cavitar – diagnostic diferențial pe RgCP între cavitate


(cavernă) și chist
Cavitate = noțiune radiologică care semnifică apariția unei arii de radiotransparență în interiorul unei
consolidări, a unei mase sau al unui nodul.
Radiologii folosesc numeroși termeni pentru a descrie ariile de radiotransprență – chist, cavitate, cavernă,
bulă, pneumatocel etc.

O abordare practică:
–Cavitate (cavernă) = radiotransparență cu perete gros (≥4mm)
–Chist = radiotransparență cu perete subțire

178. Sindromul cavitar – enumerare cauze


179. Abcesul pulmonar primitiv – cauze, patogeni
Abcesul pulmonar primitiv se dezvoltă pe un pulmon anterior indemn, datorită unei infecţii
necrotizante.
Abcesul pulmonar secundar se dezvoltă într-o leziune pulmonară cavitară preexistentă de tipul:
– Chist (hidatic sau bronhogen).
– Bulă de emfizem.
– Bronşiectazie.
– Neoplasm excavat.
– Cavernă TBC.
– Infarct pulmonar ramolit
Dezvoltarea abcesului pulmonar (indiferent de caracterul primitiv sau secundar al acestuia)
presupune şi întrunirea unor condiţii favorizante:

Deprimarea mecanismelor locale de apărare care se opun aspiraţiei


•Tulburări de deglutiţie
•Deprimarea reflexului de tuse
•Inhibarea aparatului muco-ciliar etc.

Deprimarea mecanismelor generale de apărare la


•Bătrâni
•Caşectici
•Diabetici
•Alcoolici
•După imobilizarea prelungită la pat.

180. Abcesul pulmonar – tablou clinic


Parcurge in doua stadii:
2. Stadiul de focar închis
 Este caracterizat prin simptomele şi semnele unei pneumopatii acute banale.
 Asupra riscului de abcedare trebuie să atragă atenţia
– rezistenţa febrei
– deteriorarea stării generale
– sub tratamentul antibiotic corect condus, la un pacient la care sunt prezenţi factorii favorizanţi
cunoscuţi.
3. Trecerea la stadiul de focar deschis
 Este marcată de vomică, adică de evacuarea brutală, sufocantă, a unei mari cantităţi de spută în
momentul erodării peretelui unei bronşii de către procesul septic.
 Vomica poate fi
– Masivă sau
– Fracţionată (mai frecventă astăzi), sub forma eliminării repetate a unor cantităţi de ordinul
câtorva zeci de ml de spută.
 Deschiderea abcesului într-o bronhie determină scăderea febrei şi ameliorarea pasageră a stării
generale.
 Ulterior, pacientul elimină zilnic mari cantităţi (peste 100 mL) de spută muco-purulentă, uneori
hemoptoică, cu miros fetid caracteristic (bronhoree).
 La fel de caracteristică este relaţia dintre curba febrei şi cea a expectoraţiei:
– în momentul în care expectoraţia este abundentă, febra scade datorită drenajului satisfăcător al
abcesului;
– când însă expectoraţia diminuează, febra creşte din nou
 În acest stadiu
– starea generală se menţine alterată,
– pacientul este palid şi
– pierde progresiv în greutate.

181. Abcesul pulmonar – rezultatele examenului fizic
•Este rareori caracteristic. Elementele valoroase de diagnostic pot aduce:
•Percuţia:
–Submatitate, apoi hipersonoritate unilaterală, bine delimitată (suspendată).
•Palparea:
–Vibraţii vocale exagerate (dacă abcesul e plin) sau, dimpotrivă, diminuate (dacă acesta s-a golit),
corespunzând zonei de matitate.
•Ascultaţia:
–Zgomote supraadăugate ronflante, crepitante şi subcrepitante
–Respiraţie siflantă (suflu cavernos/amforic).

182. Abcesul pulmonar – aspectul radiologic al celor 2 stadii (închis și deschis)


Radiografia toracică
–este de cele mai multe ori diagnostică. Ea furnizează imagini diferite, în funcţie de stadiul evolutiv al
supuraţiei:
•În stadiul de focar închis imaginea radiologică este cea de condensare, ca în pneumonie.
•În stadiul de focar deschis, caracteristică este imaginea hidroaerică, cu axul mare longitudinal şi
diametrul de 3-5 cm. Se poate observa un perete gros, neregulat (în ramă), întrerupt la nivelul
comunicării cu bronşia de drenaj, iar la interior, nivelul orizontal de lichid, variabil în funcţie de
eficienţa drenajului.
–Circa 1/3 din cazuri prezintă concomitent empiem pleural.

183. Abces pulmonar – evoluție, prognostic, complicații


Evolutie si prognostic:
În absenţa tratamentului, mortalitatea prin abces pulmonar se ridică la 30%, în timp ce procentul
vindecărilor nu depăşeşte 20%.Majoritatea celorlalte cazuri se cronicizează evoluând spre pioscleroză
pulmonară ireversibilă.
Sub tratament medical, bronhoreea dispare în 2-3 săptămâni, iar imaginea radiologică în 2-6
săptămâni. Sub 5% din cazuri necesită tratament chirurgical.

Complicatiile abcesului pulmonar:


Locale:
– Hemoptizie
– Bronşiectazie
– Empiem pleural sau piopneumototax
– Pioscleroza pulmonară.
Generale:
– Septicemii cu abcese la distanţă
– Anemie
– Amiloidoză secundară
– Hipocratism digital
– Caşexie

184. Tuberculoza – complexul Ghon-Ranke ( ce cuprinde ) și posibila evoluție a


primoinfecției
Tuberculoza primară (primoinfecția tuberculoasă)
–95% - infecție asimptomatică
Complexul primar (complex Ghon-Ranke)= = focar de primoinfecție + vas limfatic + ggl. limfatic asociat
are două părți:
– Tuberculom parenchimatos
– Nodul limfatic ipsilateral calcificat
Două evoluții posibile
–Infecție latentă – deși organismul poate fi infectat cu M. Tuberculosis, nu există simptome sau
semne clinice de boală. Cei mai mulți oameni prezintă sisteme imune indemne care nu vor permite
niciodată bacteriei să inducă boala.
–Vindecare
–La 5% dintre copiii infectați, imunitatea inadecvată duce la apariția unei boli active din punct de vedere
clinic - boala progresivă primară.

185. Cancerul pulmonar – definiție, tipuri


Cancerul pulmonar (bronho-pulmonar, carcinomul pulmonar) este o tumoră malignă a plămânului,
caracterizată prin creșterea necontrolată a celulelor epiteliului respirator (carcinom).
Este de două tipuri
•Cancer apărut inițial în plămâni – cancer pulmonar primitiv
•Cancerul care debutează în o altă parte a organismului, dar diseminează la plămân – cancer
pulmonar secundar.
Netratată, această creștere poate disemina în afara plămânului printr-un proces de metastazare înspre
țesuturile vecine sau din altă parte a corpului.

186. Cancerul pulmonar – tabloul clinic


•Circa 10% dintre pacienți nu prezintă simptome; în acest caz, cancerul este un diagnostic radiologic
întămplător.
Simptome
Tuse
– Prezența unei mase irită receptorii tusigeni din interiorul căilor aeriene.
– Tuse cu caracter rebel, iritativ, uscată, uneri în chinte paroxistice (adesea este vorba despre o tuse
aparent banală la un vechi fumător, care s-a accentuat în ultimul timp).
Sputa
– Mai frecventă în carcinoamele scuamoase și cu celule mici (în special în cele situate în căile aeriene
centrale).
– Obstrucția căilor aeriene centrale poate, de asemenea, determina apariția unei pneumonii pos-
obstructive sau a unei atelectazii distale.
Hemoptizia
– Apare în tumorile căilor aeriene centrale.
– Angiogeneza indusă de tumoră duce la apariția unor vase de sânge tortuoase și permeabile care se
rup cu ușurință.
– De obicei în cantitate mică (clasic descrisă ca “jeleu de coacaze”); de obicei, este simptomul care
alarmează bolnavul.
Dispneea
– Apare prin obstrucția căilor aeriene mari sau prin activarea mecano- și a chemoreceptorilor
intrapulmonari prin hipoxie/acidoză.
– Este un simptom tardiv
– Adesea, se însoțește de wheezing, care nu dispare după tuse.
Durerea toracică
– Determinată de tumorile care prind suprafața pleurală – durere pleuritică.
– Cu sediu și intensitate variabilă, întotdeauna persistentă și rebelă la tratamentul simptomatic.
Epanșament pleural (durere toracică, dispnee)
– De tip benign – prin obstrucție limfatică, pneumonie post-obstructivă sau atelectazie.
– De tip malign – celule maligne prezente în fluidiul pleural.
Scădere ponderală
– Proteoliză și lipoliză indusă de tumoră determină pierderea masei musculare și a țesutului adipos.
– La pierderea ponderală contribuie și scăderea apetitului.
Alte simptome și semne:
– Wheezing
– Greață/vărsături
– Edeme ale feței și membrelor superioare
– Oboseală
– Dureri osoase
– Hipocratism digital
– Cefalee
– Convulsii
187. Enumerați organele în care poate să metastazeze cancerul pulmonar
Cancerul pulmonar poate debuta în oricare zonă pulmonară, de unde diseminează după un pattern
predictibil.
În mod tipic, cancerul pulmonar diseminează
–Pe cale limfatică – către ganglionii cei mai apropiați, după care urmează ganglionii mediastinali.
–Prin contiguitate – către organele vecine
–Pe cale hematogenă – către creier, oase, glande suprarenale, plămânul opus, ficat, pericard, rinichi.

188. Explicați acronimul TNM


Clasificarea TNM a tumorilor maligne (TNM) este un sistem de stadializare a cancerelor solide.
– T dimensiunea tumorii primitive și dacă aceasta a invadat țesutul din apropiere.
– N implicarea ganglionilor limfatici regionali.
– M metastaze la distanță.
Are un grad mare de acceptare internațională pentru multe tipuri de tumori solide, dar nu este aplicabil
cancerelor difuze, cum ar fi cele hematologice.

189. Embolismul pulmonar: definiție, etiologie


Embolismul pulmonar (EP)- blocarea unei artere pulmonare sau a uneia dintre ramurile sale printr-un corp
străin (embol) (de obicei cheag de sânge) vehiculat prin sângele circulant în momentul în care dimensiunea
să nu-i mai permite sa înainteze.
În un mic procentaj din cazuri, embolul este format din aer, grăsimi, lichid amniotic sau alte
materiale (de ex. talc la dependenții de droguri administrate intravenos).

190. Tromboza venoasă profundă: triada Virchow, enumerare cauze


hipercoagulabilitate
Alterările componentelor sângelui– are numeroși posibili factori de risc, cum ar fi hipervâscozitatea,
mutații ale factorului V Leyden, mutații ale factorului II G2021A , deficitul de antitrombină III, deficitul de
proteină C sau S, sindromul nefrotic, modificările apărute după traumatisme sau arsuri severe, cancer,
ultimul trimestru de sarcină, naștere, rasă, vîrsta a 3-a, fumat, contracepția hormonală, obezitatea. Toate
aceste situații pot determina hipercoagulabilitate (coagularea excesiv de facilă a sângelui).

191. Tromboza venoasă profundă: triada Virchow, enumerare cauze injurie


endotelială
Leziunile și/sau traumatismul endotelialinclud penetrarea peretelui vascular, ca și leziunile consecutive
stresului de forfecare sau determinate de hipertensiunea arterială. Această categorie este dominată de
fenomene de suprafață și de contactul cu suprafețe procoagulante, cum ar fi bacteriile, apariția de
materiale străine, biomateriale din cadrul implantelor sau aparatelor medicale, membrana trombocitelor
activate, și membrana monocitelor în cadrul inflamației cronice.
192. Tromboza venoasă profundă: triada Virchow, enumerare cauze stază
venoasă
Alterararea fluxului sanguin normal– se referă la mai multe situații. Acestea includ staza venoasă,
intervențiile chirurgicale prelungite, imobilitatea prelungită (călătoriile cu avionul, imobilizarea la pat în
cursul spitalizării) și boala varicoasă. Echivalența dintre versiunea originală a lui Virchow și cea modernă a
fost disputată.

193. Manifestări clinice în embolism


Embolismul pulmonar este dificil de diagnosticat – adeseori este asimptomatic; atunci când este
simptomatic, manifestările sunt nespecifice.
Manifestările clinice variază de la lipsa completă a simptomelor la prăbușire hemodinamică catastrofală.
 Între aceste două extreme, pot apare simptome și semne variate, care pot fi grupate în mai multe
sindroame:
– Infarct pulmonar: condensare (asociată cu hemoptizie) + atingere pleurală (junghi toracic);
– Detresă respiratorie acută: dispnee;
– Instabilitate hemodinamică: hipotensiune arterială, sincopă, șoc;
– Instabilitate ritmică: bradi-/tahicardie, stop cardiac.
 Prezentarea clasică (infarctul pulmonar), caracterizată prin debut acut de durere pleuritică și dispnee
este extrem de rară (<15% din cazuri).
– Sincopa, hipotensiunea și șocul sunt semne de gravitate.
Adeseori se întâlnesc semne ale trombozei venoase profunde a membrelor inferioare (mai rar a
membrelor superioare).
În consecință, diagnosticul embolismului pulmonar trebuie suspicionat la toți pacienții cu simptome
respiratorii inexplicabile în absența oricărei alternative diagnostice.

194. Infarctul pulmonar: tablou clinic și examen fizic


Debut acut:
 Unul sau mai multe dintre urmatoarele simptome
–Dispnee
–Durere toracică de tip pleuritic
–Tuse
–Hemoptizie
 Examen fizic
–Plămâni de obicei normali;
–Semne ocazionale:
 Frecătură pleurală
 Epanșament pleural (scăderea vibrațiilor vocale, a notei percuționale, a sunetelor
respiratorii)
 Semne de suprasolicitare ventriculară dreaptă (componenta pulmonară a S2 întărită,
creșterea presiunii venoase jugulare).
 Febră/subfebră.

195. Radiografia pulmonară în embolismul pulmonar și alte teste paraclinice
(enumerare)
Radiografia toracică
– Este frecvent anormală și cu toate că aspectele radiologice, de regulă nu sunt specifice pentru EP,
metoda este utilă pentru excluderea altor cauze de dipnee și de durere toracică.
– Semnul Fleischner: creșterea în dimensiune a arterelor pulmonare, adesea asociată cu impresia de
“amputare” a arterei (20%)
– Semnul Hampton: opacitate semilunară indicănd infarctul pulmonar (20%)
– Semnul Westermark: olighemia regională – cea mai mare valoare diagnostica (10%)
– Epanșament pleural (35%)
Analiza gazelor sanguine
Electrocardiograma

Teste paraclinice de linia a 2-a:


– Dimerul D;
– Angiografia pulmonară CT;
– Ecocardiografia
– Scintigrama de ventilatie-perfuzie (V/Q scan).
– Ecografia venelor pelvine sau ale membrelor inferioare

196. Spațiul pleural: funcții


– Presiunea negativă transpulmonară reprezintă forța care menține plămânul expansionat permanent;
altfel, acesta s-ar colaba sub acțiunea reculului elastic intrinsec.
– Permite mişcarea liberă a plămânului în contact cu peretele toracic
– Permite distribuţia egală a presiunilor de inflaţie în tot parenchimul pulmonar
– Reprezintă o zonă “tampon” care preia excesul de lichid alveolar (de până la 30 de ori mai mare decât
rata normală de producere).

197. Epanșamentul pleural: definiție, tipuri ( cu explicarea mecanismului )


Acumulare de lichid în interiorul spaţiului pleural
– Liber
– Închistat (oriunde între pleura viscerală şi cea parietală sau interlobar).
– empiemul (colecție de puroi în interiorul cavității pleurale)
– hemotoraxul (colecție de sânge în interiorul cavității pleurale).
Două categorii de epanșamente pleurale:
Transudate = epanșamente cu capilare normale (rezultate din dezechilibrul fortelor Starling)
Mecanisme
– Creșterea presiunii hidrostatice în capilarele pulmonare – de ex. insuficiența cardiacă
– Scaderea presiunii coloid-osmotice în capilarele pulmonare – de ex. sindromul nefrotic, ciroza
hepatică
Caracteristici
– Fluide sărace în celule, proteine și LDH

Exudate = epanșamente cu capilare lezate (de obicei prin inflamație)


Mecanisme
– Leziuni vasculare inflamatorii infecțioase (de obicei) sau imune
– Leziuni vasculare ischemice
– Leziuni vasculare neoplazice
Caracteristici
 Fluide bogate în celule, proteine și LDH

198. Enumerați 5 afecțiuni în care epanșamentul pleural este exudativ și 5 în


care este transudativ
Epansament pleural transudativ:
 Prin cresterea presiunii hidrostatice ( cauza CV ):
– Insuficienta cardiaca congestiva;
– Supraincarcare hidro-electrolitica;
– Pericardita constrictiva;
 Prin scaderea presiunii coloid-osmotice ( hipoalbuminemie ):
– Sindrom nefrotic;
– Ciroza hepatica;
– Malabsorbtie;
– Infectie cronica;
Epansament pleural exudativ:
 Inflamatie infectioasa:
– Acuta:
 Pneumonie bacteriana => Pleurezie necomplicata => Pleurezie complicata => Empiem
– Cronica:
 Pneumonie virala;
 Vascular:
– Infarct pulmonar;
 Inflamatie auto-imuna:
– Lupus eritematos;
– Artrita reumatoida;
– Boli granulomatoase;
 Neoplazie:
– KBP;
– Metastaze

199. Epanșamentul pleural: tablou clinic


Simptomatologie
– Durere de tip pleuritic (junghi)
– Tuse neproductivă
– Dispnee
– Alte elemente de orientare etiologica:
 febră
 debut (brusc / insidios).
Anamneza trebuie să se focalizeze pe cauzele cunoscute .
200. Epanșamentul pleural: examen fizic
Examenul fizic poate decela EP în cantitate ≥ 300 ml:
–Traheea deplasată controlateral (prin deplasarea contolaterală a mediatinului)
Inspecţie
 Expansiune asimetrică a toracelui (reducerea homolaterală a expansiunii)
 ± Bombarea hemitoracelui.
Palpare
 Reducerea ampliaţiilor respiratorii
 Scăderea până la abolire a vibraţiilor vocale.
Percutie
 Submatitate deplasabilă cu poziţia.
 Curba Damoiseau-Ellis.
Auscultaţie
 Sunet pulmonar normal redus/absent.
 Scăderea transmisiei vocii.
 ±frecătură pleurală, ±suflu pleuretic.
 Percuția auscultatorie.

201. Investigații paraclinice: enumerare și descrieți aspectul epanșamentului


pleural pe RgCP
1. Radiologic
– Este nevoie de cel putin 200mL de lichid pentru ca epanșamentul pleural sa devina vizibil pe RgCP
de fata.
– Este nevoie de 50mL de lichid pentru ca epanșamentul pleural sa devina vizibil pe RgCP de profil.
– Este nevoie de 5L de lichid pentru opacifierea totala a hemitoracelui.
– Traheea și mediastinul sunt deviate ( impinse ) la dreapta.
– Dacă RgCP este neconcludentă, tomografia computerizată și ecografia pot fi utile.
2. Toracenteza - diagnostic de certitudine - orientare etiologică
3. Tomografia computerizată
– poate detecta epanșamente invizibile pe Rg
– poate face distincția între epanșamentul pleural și îngroșarea pleurei
– poate oferi indicii etiologice.
4. Ultrasonografia este mai precisă decât radiografia toracică în detectarea revărsatului pleural.
5. Pleuroscopia:
6. Toracotomia cu biopsie “deschisă”:
7. Ecografia transtoracică:
8. Bronhoscopia

202. Biopsia pleurală transtoracică: indicații și contraindicații


Biopsia pleurală transtoracică
Tehnica:
– cu ace speciale (Cope, Abrams)
– similară toracentezei, dar cu anestezie mai profundă.
– Incidente, accidente: pneumotorax, lezarea pachetului nervos intercostal, sincopa vagală.
Indicații: în orice exudat în care toracocenteza nu lămurește diagnostcul (în special pentru diferentierea TB
de neoplazie)
Contraindicații: tulburări majore de coagulare, spații i.c. foarte mult reduse.
Randament diagnostic: 80% în tuberculoză, 40-60% în neoplazii şi crește prin repetarea biopsiei.
Limite: în 7% din revarsate NU se stabileste diagnosticul etiologic nici prin biopsie → indicație de
pleuroscopie.

203. Pneumotoraxul: definiție, tipuri, etiologie


= Prezența de aer în spațiul pleural, cu colabarea în grade diferite a plămânului.
1. PTX simplu sau standard
– Pneumotorax cu volumul de aer din cavitatea pleurală constant.
– Apare caracteristic la PTX spontan la TINERII ÎNALȚI + SLABI, fumători, raport B/F=6/1, sănătoși,
prin ruptura unei bule pleurale (apărută prin defect congenital al țesutului interstițiului periferic).
2. PTX cu supapă – Atenție!!! – risc vital
– Pneumotorax cu volumul de aer din cavitatea pleurală în creștere progresivă.
– Apare prin apariția unei valve cu sens unic, care permite aerului să intre în cavitatea pleurală în
inspir, dar nu-i permite să iasă în expir.

204. Pneumotoraxul: aspect radiologic și criteriile de stabilire a dimensiunilor lui


RgCP:
–Pleura viscerală vizibilă ca o linie albă foarte subțire.
–Desenul interstițial absent în periferie.
–Plaman colabat în diferite grade → spațiul periferic radiotransparent comparativ cu plămânul
controlateral.
Împreuna cu simptomele, determinarea dimensiunii PTX ajută la stabilirea conduitei teraputice. Prin
convenție, un PTX masiv este considerat cel de >2cm la nivelul hilului.
În cazul unui pacient dispneic, sau cu o pneumopatie preexistentă, măsurile terapeutice trebuie
implementate chiar dacă PTX este de dimensiuni mici.

205. Pneumotoraxul spontan primitiv și secundar: definiție, mecanism


1. PTX primitive – fără boală pulmonară preexistentă
– Apar în mod caracteristic la adolescenți/tineri longilini – pneumotorax spontan
– Apare caracteristic la PTX spontan la TINERII ÎNALȚI + SLABI, fumători, raport B/F=6/1, sănătoși,
prin ruptura unei bule pleurale (apărută prin defect congenital al țesutului interstițiului periferic).
2. PTX secundare
– La pacienți cu boală pulmonară preexistentă – de exemplu, BPOC
– Traumatice (plăgi penetrante/nepenetrante)
– Iatrogene (ventilație mecanică, biopsii transbronșice, rupturi bronșice, catetere venoase centrale)
– Apare prin apariția unei valve cu sens unic, care permite aerului să intre în cavitatea pleurală în
inspir, dar nu-i permite să iasă în expir.
206. Pneumotoraxul compresiv: definiție, mecanism
Definiție
– Prezența de aer în cavitatea pleurală.
– Presiune intrapleurală care depășeste presiunea atmosferică, exercitând presiune asupra
structurilor mediastinale și intratoracice.
Este o situatie amenințătoare de viață și impune tratament imediat.
Fiziopatologie
– Acumulare progresivă de aer intrapleural determinată de un efect de valvă.
– Într-o asemenea situație, apar mai multe cercuri vicioase patogenice care explică pericolul vital:
 Plămânul homolateral se colabează → capacitatea vitală, ventilația/minut, schimburile
gazoase
 Venele cave sunt cudate → scade întoarcerea venoasă → scade debitul cardiac
 Mediastinul este împins și comprimă plămânul controlateral → schimburile gazoase sunt alterate
suplimentar
Diagnosticul trebuie sa fie ÎNTOTDEAUNA clinic

207. Pneumotoraxul compresiv: manifestari clinice si examen fizic


Debutul este variabil și poate debuta asimptomatic.
Simptome
–Dispnee severă, progresivă

Examen fizic
– Tahicardie, hipotensiune arterială – non-responsivă la lichide administrate i.v.
– Detresă respiratorie acută severă
– Tahipnee, cianoză
– Trahee și apex cardiac: deplasate controlateral
– Vene ale gâtului destinse
– I, P: expansiune toracică redusă sau absentă homolateral
– P: NP hipersonoră/timpanică
– A: scăderea homolaterala a sunetelor respiratorii.
– Rezonanța vocală: absentă homolateral
– Scăderea întoarcerii venoase determină stop cardiac și deces (câteodată în câteva minute)

208. Pneumotoraxul compresiv: aspect radiologic


RgCP – aceleași caracteristici cu ale unui PTX simplu, plus câteva semne adiționale.
– Colabarea plămânului tinde să fie mai evidentă
– Semne de suprapresiune în cavitatea pleurală:
 Deplasare controlaterală a traheei și a mediastinului (presiune pozitivă pe toata durata
ciclului respirator)
 Distanțare homolaterală a coastelor
 Depresie homolaterală a diafragmului.
PTX compresiv Nu trebuie ratat niciodată.
Semnele compresiei:
– Plămânul stâng este complet comprimat (săgeți).
– Traheea este deplasată controlateral (săgeată).
– Inima este deplasată controlateral – observați marginea dreaptă (linie roșie)
– Hemidiafragmul stâng este deplasat inferior (linie galbena)
ATENTIE !!! Daca diagnosticați clinic un PTX compresiv, nu mai așteptați RgCP – TRATAȚI PACIENTUL!

209. Mediastinul superior: localizare, conținut

Localizare
– între apertura toracică şi planul care uneşte unghiul lui Louis cu marginea inferioară a corpului
vertebral T4.
Conținut
– Timusul (o mică parte)
– 3 vene: brahiocefalica stângă şi dreaptă şi vena cavă superioară
– Ductul toracic
– Arcul aortic şi ramurile sale
– Traheea
– Esofagul
– Nervii frenici
– Nervii vagi
– Alte mici structuri nervoasse și vasculare

210. Mediastinul inferior: localizare, conținut:

211. Sindroamele mediastinale: definiție, enumerați cauze responsabile de


apariția lor
Sindroamele mediastinale sunt anomalii ale mediastinului simptomatice, maligne.

Anomalii ale mediastinului


•30% sunt tumorale
•95% sunt benigne
•5% sunt maligne
•70% sunt non-tumorale
Multe afecţiuni mediastinale sunt asimptomatice sau cu o evoluţie foarte lenta

În funcţie de localizarea anatomică şi natura procesului patologic cauzal pot aparea:


1.Manifestări de compresie / invazie traheo-bronşică
2.Manifestări de compresie vasculară şi limfatică
3.Manifestări de compresie nervoasă
4.Manifestări de compresie / invazie esofagiană
5.Manifestări de ordin general.

212. Sindroamele mediastinale: definiție, manifestări de ordin general


– Manifestări de ordin general:
– Hipo- / hipertiroidism (gusa mediastinala)
– Hipoglicemie (tumori mezenchimale)
– Diaree (ganglioneurinoame, neuroblastaome)
– Miastenia (timoame)
– Anemie aplastică pură (timoame)
– Hipogamaglobulinemie (timoame)
– Sindrom Cushing (timoame, carcinoid)
– Ginecomastie (tumori germinale)
– HTA (feocromocitom, ganglioneurinoame)
– Hipercalcemie (limfoame, tumorile paratiroidei)
– Dureri la consumul de alcool (boala Hodgkin)

213. Sindroamele mediastinale: enumerați investigațiile paraclinice și argumentați


importanța CT-ului.
1. RgCP (față + profil)
2. Tomografia computerizata
– Pune diagnosticul în peste 90% din sindroamele mediastinale.
– In momentul actual, este considerata cea mai buna tehnica de explorare a meselor toracice,
deoarece:
 Poate vizualiza formatiuni de mici dimensiuni.
 Poate localiza cu precizie leziunile și raporturile acestora.
 Poate identifica compoziția tisulară a leziunilor.
3. Testele finale:
–Pentru obținerea biopsiei – mediastinoscopia cervicală, toracotomia exploratorie.
–Pentru tumorile mediastinului posterior – MRI (la acest nivel, majoritatea tumorilor sunt neurogene).
–Pentru o masă nedefinită tomografic – PET-CT (suspiciune înaltă de leziune malignă).

214. Insuficiența respiratorie: definiție, valori PaO2, PaCO2, PA-aO2 în hipoxemia fără
hipercapnie, cauze ( pentru acest tip de insuficiență respiratorie )
Sindrom definit prin schimburi gazoase inadecvate la nivelul aparatului respirator, având ca rezultat
incapacitatea de menținere în limite normale a PaO2, a PaCO2 sau a ambelor.
 Scădere a PaO2 sanguin = hipoxemie N > 80mmHg (>11kPa)
 Creștere a PaCO2 sanguin = hipercapnie N < 45mmHg (6kPa)
Tipul 1 = hipoxemie fără hipercapnie (I.R. pulmonară, I.R. hipoxemică, insuficiența de oxigenare)
Cauze
 Scăderea oxigenului ambient (altitudine inalta)
 Dezechilibru ventilatie/perfuzie (V/Q) – cea mai frecventă cauză
– Părți ale plămânului primesc aer, dar nu primesc sange (de ex. embolismul pulmonar)
 Hipoventilație pulmonară (scăderea minut-volumului din cauza reducerii activității mulchilor
respiratori, de ex. în bolile neuro-musculare acute) – această situație poate determina și insuficiență
rspiratorie de tip II (în cazul în care este severă).
 Probleme ale difuziunii (pneumonie, pneumopatii interstițiale, ARDS)
 Șunt (sângele oxigenat se amestecă cu cel non-oxigenat din sistemul venos, de ex. în șuntul dreapta-
stânga)
PaO2 Scăzut (<60mmHg)
PaCO2 Normal (<50mmHg)
PA-aO2 Crescută

215. Insuficiența respiratorie: definiție, valori PaO2, PaCO2, PA-aO2 în hipoxemia cu


hipercapnie, cauze ( pentru acest tip de insuficiență respiratorie )
Sindrom definit prin schimburi gazoase inadecvate la nivelul aparatului respirator, având ca rezultat
incapacitatea de menținere în limite normale a PaO2, a PaCO2 sau a ambelor.
 Scădere a PaO2 sanguin = hipoxemie N > 80mmHg (>11kPa)
 Creștere a PaCO2 sanguin = hipercapnie N < 45mmHg (6kPa)

Tipul 2 = hipoxemie plus hipercapnie (I.R. ventilatorie, insuficiență de pompă, I.R. hipercapnică)

Cauze – hipoventilație alveolară


– Afectarea SNC (medicamente, leziuni ale trunchiului cerebral)
– Afectarea SNP (boală a neuronului motor, de ex. sindromul Guillain-Barré)
– Slăbiciune a mușchilor respiratori
– Defecte ale peretelui toracic (cifoscolioza, spondilita ankilopoietică, obezitatea extremă)
– Obstrucție a căilor aeriene (BPOC, astm bronșic).
PaO2 Scăzut (<60mmHg)
PaCO2 Normal (<50mmHg)
PA-aO2 Crescută
pH Scăzut (acidoza respiratorie)

216. Clasificarea practică a insuficienței respiratorii


E de doua tipuri:
 Hipoxemica (insuficienta de oxigenare) PaO2 <60mmHg, oxigenoterapie 60%:
– Acuta (minute-ore);
– Cronica (zile-mai mult);
 Hipercapnica (insuficienta ventilatorie) PaCO2 >45mmHg, pH<7.35:
– Acuta (minute-ore)
– Cronica (zile-mai mult)
217. Apneea nocturna: definitie, etiologie
Tulburările respiratorii asociate somnului cuprind mai mult condiții, de obicei asociate cu
îngustarea/obstrucția căilor aeriene superioare (faringe).

1. Apnee nocturnă obstructivăse referă la obstrucția intermitentă a faringelui în timpul somnului,


indiferent de prezența simptomelor diurne (85%).
2. Apneea nocturnă centralăîntreruperea periodică a respirației survine fără obstrucția căilor aeriene; la
adult, apare mai ales la pacienții cu insuficiență cardiacă (0.5%).
3. Apneea centrală de tip mixt (15%)

218. Apneea nocturna: Patogenie și fiziopatologie


Simptomele apneei nocturne sunt clasificate ca nocturne (în timpul somnului) și diurne.Cel mai
frecvent simptom este somnolența diurnă. Nu apare la toți pacienții.

În timpul somnului
– Sforăit sonor
– Apnee afirmată de partenerul de somn
– Trezire cu senzație de sufocare
– Agitație nocturnă
– Vise vii, stranii, amenințătoare
– Reflux gastro-esofagian
– Insomnie cu treziri frecvente
– Nocturie (urinat nocturn)
– Hipersalivație
– Diaforeză (transpirații) nocturnă

În timpul zilei
– Somnolență diurnă
– Impresia de somn neodihnitor
– Lipsă de concentrare
– Modificări de dispoziție
– Cefalee matinală
– Gură uscată
– Impotență sau libidou scăzut

219. Apneea nocturna: manifestari clinice si diagnostic


Diagnosticul nu este întotdeauna simplu; mulți oameni sforăie noaptea și au somnolență diurnă, dar
niciunul dintre aceste simptome nu este foarte specific.
Trebuie eliminate:
– Sforăitul simplu
– Privarea de somn
– Depresia
– Hipotiroidismul
– Boala de reflux gastro-esofagian – care determină episoade de tuse nocturnă
Polisomnografia
În mod tradițional, este standardul de aur al testelor diagnostice. Metoda exactă de testare variază, dar, de
obicei, polisomnografia monitorizează:
– Fluxurile aeriene nazal și/sau oral
– Mișcările toraco-abdominale
– Sforăitul
– Electroencefalograma (EEG) pentru măsurarea activității undelor cerebrale
– Electrooculograma (EOG) pentru măsuarea mișcărilor globilor oculari
– Electromiograma (EMG) pentru măsurarea activității musculare
– Saturația de oxigen
Este costisitoare,deranjantă pentru pacient și impune ca pacientul să rămână peste noapte în spital.

220. Apneea nocturna: complicatii


Cognitive
–Probleme de memorie
–Somnolență diurnă
–Incapacitate de concentrare
–Creșterea riscului de accidente – de exemplu, în timpul condusului
–Iritabilitate
–Modificări de personalitate

Cardio-vasculare
–Hipertensiune pulmonară
–Cord pulmonar
–Hipertensiune arterială
–Risc crescut de boală cardiacă ischemică
–Risc crescut de aritmii cardiace

Altele
–Risc crescut de accident vasxcular cerebral
–Risc crescut de mortalitate de orice cauză, indiferent de factori de risc descriși mai sus
–Risc crescut de diabet zaharat tip 2

221. Sindromul de obezitate – hipoventilatie: definitie, clasificare

Combinație de:
– Obezitate (BMI >30 kg/m2 ),
– Hipoxemie nocturnă,
– Hipercapnie diurnă,
Determinate de hipoventilație (respirație insuficient de rapidă sau insuficient de profundă).

Mulți dintre acești pacienți (90%) prezintă întreruperi nocturne ale respirației (apnee nocturnă).Această
afecțiune suprasolicită cordul drept, ceea ce duce în cele din urmă la cord pulmonar.

Sunt recunoscute 2 subtipuri, în funcție de natura alterării respiratorii detectate la investigațiile


amănunțite:
 SOH în contextul apneei nocturne (90%); aceasta este confirmată în contextul a 5 sau mai multe
episoade de hipopnee-apnee pe oră (index apnee-hipopnee) în timpul somnului.
 SOH determinată de “sindromul hipoventilației nocturne” (10%); aceasta necesită un nivel sanguin al
CO2 după somn cu 10mmHg mai mic comparativ cu măsurătorile efectuate în curs de stare de veghe, în
absența episoadelor nocturne de apnee-hipopnee.

Cel mai eficient tratament este scăderea ponderală.

222. Sindromul de obezitate – hipoventilatie: manifestari clinice si diagnostic


Două tipuri de manifestări
–Apnee de somn de tip obstructiv
–Simptome și semne de forțare a cordului drept (cor pulmonale).
Criteriile de diagnostic:
–BMI >30 kg/m2
–PaCO2 >45 mmHg (>6.0 kPa)
–Absența unei explicații alternative pentru hipoventilație (utilizarea narcoticelor, disfuncții ventilatorii
obstructive sau restrictive, modificări sevefe ale peretelui toracic, cum ar fi cifoscolioza, hipotiroidism
severf, boli neuro-musculare).

Вам также может понравиться